SlideShare une entreprise Scribd logo
1  sur  128
Télécharger pour lire hors ligne
Дэд проф Ц.Сумъяа




Анrли хэл
              егуулбэр зуй
         (Дурэм, да.сгал, сорихго)
            Гурав дахь хэвлэл




Хянан тохиолдуулсан Др.Проф. Чой.Пувсснжов




               Улаанбаатар
                   2004
Chapter Eight Exercises: Exercise

  3. Through the use of computers and modems to send data over
     telephone lines, efficient communications is possible.
  4. "Modulator and demodulator" are what modem stand for.
 5. Thousands of electronic "bulletin boards" across the country is
     actually computers with modems that exchange data.
 6. Thanks      is due to volunteers called SYSOP's, or system
operators,
     for operating those bulletin boards.
  7. A network of users asks questions and checks the bulletin board.

  8. Graphics are posted for others to copy for their own use.
  9.    A team of computer operators use modems to transmit
messages
     at different speeds.
 10. At any speed, tvelve pages take less time to send by modem
than
     by even the speediest mail service.


             XOpMH    rypaanyraap Aacran Exercise Twenty Three
 ,QapaaXb erVV1l63PVVAI.1Vir   H5IrTIlaH yHwaaA, 6yxl.1Vi II anaaar
sanpyynas 61.1'-1. 8ryyJ13rAXVVH, eryyJ13xVVHI.1Vi 30X1.14011A ronnoa
asxaapax.

       A dazed crowd wander about staring at the debris. Alone on the
boards of the second floor stand a toilet and sink a tub sits firmly on a
concrete slab with the house in rubble around it. Two pickup trucks in
front of the house rocks with their wheels in the air. The next house
don't have a roof but the house after that is untouched.
       This sene debris and chaos are a disaster. A tomaedo has
struck. No one has been killed, but hundreds of people is stunned.
Fortunately, there are some relief on the way. How do the Red cross
help? Within a day, a crew of caseworkers begin to interview the
families and porvide assistance. Whatever the victims doesn't have
any longer- clothes, as well as food and medicine- is donated and
distributed.
The Red Cross distribute money and vouchers rather than actual
goods because that helps the local economy. Many businesses
appears damaged by the disaster, and they needs customers to help
them rebuild. Politics play no part here; what the Red Cross provides
are help and support for all.



                                                                       75
ChapterEight Exercises: Exercise

                       H3r A3X     copanro Test One
Directions: One or more of the underlined sections in the following
sentences may contain errors grammar, usage, punctuation, spelling, or
capitalization.

Write the letter of each incorrect section, then rewrite the item correctly. If
there is no error in an item, write E.

             Example According to Charles L. Hogue of the .=:Lo~s~_
                                        A

             Angeles county Musuem of Natural Historv. "Knee-high
             B                                 C

            to a grasshopper" is about one-half an inch. No error
                                                        D        E
            Answer B. Los Angeles County C. History, " knee-high


1.Thomas Jefferson wrote the Declaration of Independence. Which was later
            A                                                B
revized by Benjamin Franklin, John Adams, Congress. and Jefferson
himself.
    C                                                       D
No error
    E

2. The word anesthesia coined in 1846 in Boston, was formed from greek
                A              B
word parts meaning "lack of feeling," No error
                        C      D        E
3. Aspirin is the most widely used drug in the world. scientists still do not
      ABC                                            D
know why or how it works. No error
                                  E
4, Santa Fe's official name Villa Real de la Santa de Fe San Francisco Asis
        A                           B
is one of the longest place names in the world, No error
                  C                        D      E
5. Our sixteenth president Abraham Lincoln was voted the best U.S.
                              A
President in a 1982 survey of leading historians. political scholars and
              B C D
authors. No error
               E

76
Chapter Eight Exercises: Exercise

6. Originaly written as a bank commerical. "We've Only Just Begun" became
        A B C
a popular hit in the 1970's. No error
                        D           E
7.Plays. written by William Shakespeare are performed throughout the world
                          A                                                B
in such diverse languages as Japanese, Russian, , Italian and Hebrew. No
                      C                                    D
error
    E

8. The twentv first century which will begin with the year 2001 is almost upon
             A B C
us. No error
D           E
9. Although Poetry magazine was only a small Chicago periodical it
                A                                                 B
contributed to the early recognition of such great poets as Frost. Pound.
Eliot,
                                                 C
Sandburg. and Edgar Lee Masters. No error
        D                                E
10. While some people argue about whether the Loch Ness monster exists
&
                            A                            B
all. others think there may be more than one. No error
    C               D                                E

11. President Truman's full name was Harry S Truman the "S" was not an
                A                                        B
abreviation for a name and was correctly written without a period. No error
      C                               D                             E

12. Although Othello is the title role in the play of that name. The vilJian lago
                                   ABC
has almost half again as many lines. No error
                                D        E




                                                                               77
Chapter Eight Exercises: Exercise

Directions Each of the following sentences has a grammatical
problem. On your paper write the letter of the corrected sentence.

13. The Baghdad battery is an an articraft that looks like a modern
chemical battery found in the ruins of a 2000-year-old village.
     A. The Baghdad battery is an articraft that looks like a modern
     chemical battery found. In the ruins of a 2000-year-old village.
     B. Found in the ruins of a 2000-year-old village, the Baghdad
     battery is an articraft that looks like a modern chemical battery.
     C. The Baghdad battery is an articraft that looks like a modern
     chemical battery to find in the ruins of a 2000-year-old village.

14. Asking a price of $200000 in 1980, a tropical rain forest near Sea

Turtle Park in Costa Rica was offered for sale.
      A. A tropical rain forest near Sea Turtle Park in Costa Rica was
      offered for sale, asking a price of $ 200 000 in 1980.
      B. Asking a price of $200 000, a tropical rain forest near Sea
      Turtle Park in Costa was offered for sale in 1980.
      C. Asking a price of $ 200 000, in 1980 a government agency
      offered a topical rain forest near Sea Turtle Park in Costa Rica
for sale.

15. Appearing in 1894, Joseph Pulitzer published the first full-colur
comic strip in his New York newspaper.
      A. Appearing in 1894 in his New Your newspaper, Joseph
      Pulitzer published the first full-colour comic strip.
      B. Joseph Pulitzer published the first full-colour comic strip in
his . New York newspaper appearing in 1894.
      C. Appearing in 1894, the first full-colour comic strip was
      published by Joseph Pulitzer in his New York newspaper.

16. To put astronauts on the moon, many hours of scientific study
and vast amounts of money were expended.
     A. Putting astronauts on the moon, many hours of scientific
     study and vast amounts of money were expended.
     B. To put astronauts on the moon, NASA expended many hours
     of scientific study and vast amounts of money.
     C. To put asronauts on the moon, vast amounts of money and
     many hours of scientific study had to be expended.




78
Chapter Eight Exercises: Exercise


  17. One can see the backward-growing hair that gives a Rhodesian
  ridgeback its name along its spine.
        A.     The backward growing hair that gives a Rhodesian
  ridgeback
        its name can be seen along its spine.
        B. Along its spine, the backward-growing hair that gives a
        Rhodesian ridgeback its name one can see.
        C. The backward-growing hair that one can see gives A
        Rhodesian ridgeback its name along its spine.
  18. Alarmed by the mild tremors, supplies were hoarded and survival
  kits were bought.
        A. Supplies, alarmed by mild tremors, were hoarded and survival
        kits were bought.
        B. Being alarmed by the mild tremors, supplies were hoarded
  and survival kits were bought.
        C. Alarmed by the mild tremors, people hoarded supplies and
        bought survival kits.

         CopMIlro          Test 2

  Directions One or more underlined sections in the following sentences
  may contain errors of grammar, usage, punctution, spelling or
  capitalization. Write the letter of each incorrect section; then rewrite
  the item correctly. If there is no error in an item, write E.

               Example      The first money to carry the motto" In God We
                                      A
  B                    .
               Trust" ~ bank notes which apeared in 1864. No
                         C                      D
               error
                 E
              Answer C -was D appeared


1.Warner Brothers studios must have wanted to have set a record when it
                  A                        B
asked John Barrymore to bestow 191 kisses on a number of beautiful
               C
senoritas in the film Don Juan. No error
                           D              E


                                                                        79
Chapter Eight Exercises: Exercise

2. Before his death in 1941, Robert Baden-Powell, who founded the Boy
                                  A
Scouts, is also a British intelligence officer. No error
  BCD                                                E
3. Many of the workers building the St. Gotthard Tunnel in Switzerland in
    A B C
1882 were injured. No error
             D                E
4. The lute. with its descendents the sitar, violin, fiddle, guitar, and
ukulele,
              A
are more widely used than any other stringed instrument in the world. No
B C D
error
     E

5.Nether a large handful of potato chips       D.Q[   a large bowl of pretzel have
as
                          A                    B                              C
much salt as one serving of canned soup. No error
                   D                                    E
6. If Vincent van Gogh would have been able to sell his paintings for what
                                          A
they bringtoday.he would have died a billionaire several timesover instead
           B C D
of a pauper. No error
                      E
7.        The sphinx, a beautiful and mysterious statue,    se~   not far from where

                  A                                         B
the Great Pyramid of Giza raises into the sky. No error
               C             D                    E
8. Though they could have fled before the storm began, Lee Trevino, Jerry

                        A                   B
Heard, and Bobby Nickols were all struck by lightning at the Western
Open
                                  C                          D
Gold Tournament in 1975. No error
                                      E



     80
Chapter Eight Exercises: Exercise

9. For camouflage a bug called Fulgora lucifera has a false head that
lays
           A B C
on the ground and resembles the head of a South American alligator. No
                      D
error
  E

10. Truth or Consequences were first seen in 1956, making it one of the
                             ABC
oldest game shows on television. No error
      D                                   E
11.       Native Americans had been living in what i§. now the United States
for
                               ABC
thousands of years before Christopher Columbus "discoered" it. No error
                                              D                         E
12. Although virtually every man and woman in America are familiar with

                      A                                            B
the ice-making properties of the freezer, few is aware that this mashine was
                                                  C
invented by a doctor to provide ice for fever-racked patients. No error
                                              D                     E
13. The rattlesnake and the copperhead is famous for being poisonous.
but
                                           A                          B
the cobra is the most dangerous venomous snake. No error
                       C              D               E
14. In the television series,''The Many loves of Dobie Gillis," a whole cast
of
                                                A
Futurestars, includingWarren Beatty and Tuesday Weld, were representative
                  B                                                 C
of average American teenagers. No error
                    D                     E
15. Anyone who saw the film Dr,X, has seen Humprey Bogart in a horrer
                ABC                                            D
movie. No error
              E
16.Marlon Brando was bom in Omaha Nebraska,but he don't sound like a
            .         A              B            --C
Midwesterner in The Godfather. No error
                          D           E
                                                                               81
Chapter Eight Exercises: Exercise

17. CaseyJones is one of thefew American folk heroswho was real and not
                                       ABC
simply created by a songwriter or storyteller. No error
                              D                 E
18. There § absolutely no "man-eating" plants in the world, although some

            ABC
plants do trap insects.No error
       D                  E
19. Locusts. whichcome out of the groundonce everyseventeen years, is a
         A B C                                                   D
serious problem in some areas of the Midwest. No error
                                                 E
20.The American colonists declared there independance from Great
Britain
                                        A           B
only after they had tried every other means to gain justice. No error
     C              D                                         E
21. Giant hailstones weighing more than a pound each begun to kill
cattle
                            A                            B
as they failed on the Russian village of Kostov in July 1923: twenty-three
           C                                                 D
people were killed trying to save the animal. No error
                                                E
22.DonQuixotemounted his horsewhile his guire, SanchaPanza, holds the
                         ABC                                       D
animal steady.No error
                  E
23. of one hundred crossword puzzle creators who contributed to two
New York puzzle magazines in1970. one-fourth ~in prison. No error
  B C D                                                       E
24. Neither the members of the city council nor the chairperson~
                     A B C
available for questions from the press. No error'
                                  D         E




  82
Chapter Eight Exercises: Test


copanro A

8ryyn63p yycr3x3A     3a~Ilwry~ xspsr 6oIlAor yrrnarar X3114, 3C
ryxaanax X3Il4, Y~IlTH3PIo1~H 60IlOH aexuerrr yrlo1~H X3Il4, H3P rlo1wyyH
eryyIl63p,  Y~Il xascpan 6ylOy 6a~4 rlo1wyyH eryyIl63p rlo1wyyH,
TOAOTrOIl,AaraIlAax TOAOTrOIl, MeH T3p4Il3H eryyIl63plo1~H yrlo1~H A3C
napaa 33pr33p repenxces copanrsir 3H3 X3C3rT opyynas. COPIo1Ilro
6YPIo1~r xoneornox AYP3MT3~ Ys:lIlAYYIlaH a)f(Io1IlIla)f( napaa Hb eryyIl63p
6YPIo1~r caarap Hs:lrTIlaH, MOHrOIl X3IlHIo1~X33 eryyIl63p 6yr3x 3Y~ Aaryy
Ha~PYYIlaH op-tyynax Hb TaHbl aHrIllo1 X3Il 60IlOH 3X X3IlHIo1~ M3AIl3rT
H3H rycraa 60IlHO.

CopMnroA-1


Task: Completing structure problems involving incomplete
independent clauses.
Directions: Choose one out of A,B,C,and D to complete the sentence.

1. In the United States,             is generally the responsibility of
municipal
     govemments.
     A. for water treatment         B. water treatment
     C. where water treatment              D. in which water treatment

2. Crop rotation        of preserving soil fertility.
    A. it is one method                B. one method
      C. a method is one            D. is one method

 3.          the dollar as its monetary unit-in 1878.
      A. Canada adopted            B. Adopted by Canada
      C. It was adopted by Canada        D. The Canadian adoption of

4.          almost impossible to capture the beauty of the aurora
borealis
     in photographs.
     A. Being                     B. It is
     C. There is                  D. Is

;5. Usually political cartoons           on the editorial page of a
newspaper.
    A. appear                        B. whose appearance
    C by appearing                   D. when they appear

                                                                          83
Chapter Eight Exercises: Test


6.             two major art museums, the Fogg and the Sadler.
      A.     Harvard University has B. At Harvard University
     C       Harvard University, with its D. There at Harvard
University

7. American actress and direstor Margaret Webster       for her
production
     of Shakespearan plays.
     A. who became famous            B famous as she became
     C becoming famous          D became famous

8.              gas tanks connected to welding equipment, one of full of
oxigen      and the other full of acetylene.
      A. It is two                        B. Of the two
      C. There are two                    D. Two

9. --,-,,..--_ is more interested in rhymtn than in melody is appearant
from his      compositions.
      A. That Philip Glass             B. Philip Glass, who
      C. Philip Glass                  D. Because Philip Glass

10. Compressed air              the power to drive pneumatic tools.
     A. by providing                B. provides
      C. that provides              D. the 'provision of

11.              by cosmic rays.
      A. The Earth is constantly bombarded
    B. Bombarded constantly, the Earth
    C. Bombarding the Earth constantly
    D. The Earth's constant bombardment
 12            primary colors are red, blue,and yellow.
    A. There are three                 B. The three
    C. Three of them                   D. That the three

13. =-               who was elected the first woman mayor of Chicago
in 1979.
      A. It was Jane Byrne         B. Jane Byrne
      C. That Jane Byrne                D. When Jane Byrne

14. Every computer consists of a number of systems -'-__
together.
      A...by working            B. work
      C. they work                   D. that work

84
Chapter Eight Exercises: Test

 15. On the Moon,             air because the Moon's gravitational field
is too weak to retain an atmosphere.
      A. there is no                    B. where no
       C. no                            D. is no

 16. The Glass Mountains of northwestern Oklahoma                   .with
flecks of gypsum, which shine in the sunlight.
      A. they are covered       B. covered them
      C. that are covered       D. are covered

 17. In some cases,              to decide if an organism is a plant or
an animal.
     A. difficult if                     B. it is difficult
     C. the difficulty              D. is difficult

18. The first American novelist to have a major impact on world
literature        _
       A. who was James Fenimore Cooper
       B. James Fenimore Cooper was
       C. it was James Fenimore Cooper
       D. was James Fenimore Cooper

  19.         important railroad tunnel in the United States was cut
through the Hoosac Mountains in Massachusetts.
      A. At first                      B. It was the first
      C. The first                            D. As he first of
20. ·Generally,            in the valleys and foothills of the Pacific Coast
ranges
      A. the Calfornia poppy grown            B. the growth of the Calfornia
poppy
      C. the Calfornia poppy grows D. growing the Calfornia
poppy
21. When bats are at rest,               hang upside-down.
      A. they     B.and         C. to         D.as
22.            that the capital of South Carolina was moved from
Charleston to
      Columbia.
      A. in 1790 was                          B. There was in 1790
      C. in 1790                              D. It was in 1790
23. Although not as important as they once were, _ _ a major form
of transprotation in North America.
      A. there are still railroads            B. railroads, which are still
      C. railroads are still                  D. railroads still being


                                                                            85
Chapter Eight Exercises: Test

24. The loop, which is the commercial heart of Chicago,               _
within a rectrancular loop of electrical train tracks.
      A. that is enclosed                    B. enclosing
      C. is enclosed                         D. it is enclosed
25. _....,.--,....- amino acids that serve as the basic building blocks of all
      proteins.
      A. It was about twenty                 B.Forabouttwentyof
      C. About twenty are                    D. There are about twenty

CopMnro A-2

Task: Answer structure problems involving incomplete adjective
clause
Directions: Choose one option out of A,B,C, and Dand encircle the
correct word or phrase.

1. Most folk songs are ballads               have simple words and tell
simple stories.
     A. what                                B. although
     C. when                                D. that

2. After its introduction in 1969, the best float process        the
world's principal method of manufacturing flat sheets of glass.
     A. by which it became                       B. it became
      C. became                                  D. which became

3. In 1850, Yale university established Sheffield Scientific School,

      A.   engineers were educated there
      B.   where engineers were educated
      C.   in which were engineers educated
      D.   where were engineers educated

 4. Many of Louse Nevelson's sculptures consisted of a number of
large wooden structures       in complex patterns.
      A. which she arranged               B. she arranged them
      C. which arranged                   D. arranged them

5. In addition to being a naturalist. Stewart E. White was a writer
_ _ the struggle for survival on the American Frontier.
    A. whose novels describe B. he-describes in his novels
     C. his novels describe        D. who, describing in his novels



86
Chapter Eight Exercises: Test

6. Diamonds are often found in rock formations called pipes, _ _
the throats of extinct volcanoes.
      A. in which they resemble        B. which resemble
      C. there is a resemblance to     D. they resemble

7. William Samuel Johnson,              helped write the Constitution,
became    the first president of Columbia College in1787.
    A. whom he had                 B. and he had
     C. who had                    D. had


 8. Seals appear clumsy on the land,         are able to short
distances faster
      than most people can run.
      A. but they                B. which they
      C. they                    D. which

 9. The instrument panel of a light airplane has at least a dozen
instruments
      A. the pilot must watch
      B. what the pilot must watch
      C. which the pilot must watch them
      D. such that the pilot must watch them

 10. A keystone species is a species of plants or animals _ _
absence has      a major effect on an ecological system.
     A. that its                   B. its
     C. whose                      D. with its

 11. The size and shape of a nail depends primarily on the function
          in-
----:t:-ended.
        A. which it is            B. for which it is
        C. which it is for        D. for which is

12. In geometry, a tangent is a straight line     a curve at only
one point.
     A. it touches                B. whose touching
     C. its touching              D. that touches

13. It was the ragtime pianist Scott Joplin       the Maple Leaf
Rag, perhaps the best known of all ragtime tunes.
     A. wrote                    B. the writer of
     C. who wrote                D. writing

                                                                         87
Chapter Eight Exercises: Test

 14. There are 2,000 varieties of snakes, _ _are harmless to
humans.
     A. mostly they               B. most of them
     C. most of which             D. which most

15. Smokejumpers are _ _ descend into remote areas by
parachute to fight    forest fires.
     A. firefighters                B. when firefighters
     C. who, as firefighters        D. firefighters who

16. Charlotte Gilman's best known book                she urges women
to  become financially independent.
    A. is Women and Economics, in which
    B. Women and Economics, in which
    C. is Women and Economics, which
    D. Women and Economics, which

CopMnroA·3

Task: Structure problems connected with incomplete or missing
participial phrases.

 Directions: Choose the one option out of the A,B,C, and D to
complete the sentence correctly.


 1. Aerodynamics is the study of the forces              on an object as
it moves through the atmosphere.
      A. acting                        -B. act
      C. are acting                     D.. acted

2.             for their strong fiber include flax and hemps.
      A. Plants are grown                  B. Plants grown
      C. Plants that grow                  D. To grow plants

3.             ,Jose Limon's dance troupe often toured abroad.
      A.   The U.S. State Department sponsored it
      B.    Sponsored by the U.S.State Department
      C.    The U.S. State Department, which sponsored it
      D.    The sponsorship of the U.S. State Department

4. Elfreth's Alley in Philadelphia is the oldest residential street in the
United States, with         from 1728.
     A. houses are dated                  B. the dates of the houses
     C. the dating of houses              D. houses dating
88
Chapter Eight Exercises: Test


5. In,1821, the city of Indianapolis, Indiana, was laid out in a design

      after that of Washington,D.C.
      A. patterned                              B. was patterned
      C. a pattern                              D. that patterned

 6.                  in front of a camera lens changes the color of the
light that reaches the film.
       A. Placed a filter                   B. a filter is placed
       C. A filter placed                   D. When a filter placed

7. The Masschusetts State House,              in 1798, was the most
    distinguished building in the United States at that time.
    A. completing                      B. which was completed
    C. was completed                   D. to be completed

8. Barbara McClintock               for her discovery of the mobility of
generic elements.
     A. known                             B. who knows
     C. knowing                           D. is known

9. The solitary scientist         by himself has in many intances
been replaced by a cooperative scientific team.
     A. to make important discoveries
     B. important discoveries were made
     C. has made important discoveries
     D. making important discoveries

 10. Geometry is the branch of mathemaics            the properties
of lines,
       curves, shapes, and surfaces.
       A. that concerned with             B. it is concerned with
       C. concerned with                  D. its concerns are

11.             an average of 471 inches of rain a year, Mount
Waiaeale in     Hawai is the wettest spot in the world.
    A. It receives                     B. Receiving
    C. To receive                      D. Received

12. Amber is a hard, yellowish-brown               from the resin of pine
trees      that lived millions of years ago.
      A. substance formed                 B. to form a substance
      C.. substance has formed            D. forming a substance

                                                                           89
Chapter Eight Exercises: Test


CopMnroA-4

Task: Structure problems with appositives.
Directions: Choose one option that correctly complete the sentence.
Mark the most appropriate one out of A,B,C, and D.

 1. The Democratic party is older than the other major American
political party

      A.   which the Republican party
      B.   the Republican party
      C.   it is the Republican party
      D.   the Republican party is

2.            relations with friends and acquaintances, playa major
role in the social development of adolecents
       A. What are called peer group relations are
       B. Peer group relations are
       C. Peer group relations, the
       D. By peer group relations, we mean

3. Joseph Henry,           director of the Smithsonian Institution,
was President Lincoln's adviser on scientific matters.
    A. the first                        B. to be the first
    C. was the first                   D. as the first

4. The Wasstch Range,            extends from southeastem Idaho
into northem Utah.
     A. which is a part of the Rocky Mountains,
     B. a part of the Rocky Mountains that
     C. is a part of the Rocky Mountains
     D. a part of the Rocky Mountains, it

5.           Ruth St.Dennis turned to Asian dances to find inspiration
for her     choreography.
      A. It was the dancer              B. The dancer
      C. That the dancer                D. The dancer was

 6. The organs of taste are the _ _ that are mainly located on the
tongue.
     A. groups of cells,are taste buds
     B. taste buds. are groups of cells
     C. taste buds, these are groups of cells

90
Chapter Eight Exercises: Test

      D. taste buds, groups of cells

7. In 1878, Frederick W.TayJor invented a concept called scientific
     management,          of obtaining as much efficiency from
workers and machines as possible.
     A. it is a method               B. a method which
     C. a method                     D. called a method

8. A group of Shakers, __settled around Pleasant Hill, Kentucky,in
1805.
     A. members of a strict religious sect which
     B. whose members of a strict religious sect
     C. members of a strict religious sect,
     D. were members of a strict religious sect

9. In physics,          "plasma" refers to a gas that has a nearly
equal      number of positively and negatively charged particles.
     A. the term                       B. by the term
      C. is termed                     D. terming

10. Norman Weiner,           mathematician and logician, had an
important role in the development of the computer.
     A. who, as a                      B. was a
     C. whom a                         D. a

 11. Jeroma Kern's most famous work is Showboat,             most
endUring
     musical comedies.
     A. it is one of the finest,     B. one of the finest,
     C. the finest one               D. as the finest of the

 12.         a marshland that covers over 750 square miles in North
Carolina and Virginia.
A. In the Great Dismal Swamp B. The Great Dismal Swamp, which
C. The Great Dismal Swamp,      D. The Great Dismal Swamp is


copanro- A-5


Task: Completing structure problems involving adverb clauses,
reduced adverb clauses, and prepositional expesssions.
Directions: Choose the one option out of A,B,C, and D incircling the
most appropriate one

                                                                       91
Chapter Eight Exercises: Test


1. Small sailboats can easily capsize           they are not handled
carefully.
     A. but           B. which        C. if          D. so

 2.         they are tropical birds, parrots can live in temperate or
even cold climates.
     A. Despite       B. Even though C. Nevertheless D. But

3.          added to a liquid, antifreeze flowers the freezing
temperature of that liquid.
    A. That    B. As is     C. It is          D. When

4.          advertising is so widespread in the United States, it has
had an    enormous effect on American life.
      A. Why B. The reason C. On account ofD. Since

 5.            toward shore, its shape is changed by its collision with
the   shallow sea bottom.
      A. During a wave rolls            B. As a wave
      C. A wave rolls                   D. A wave's rolling

6. ____ are increasingly linked over long distances by electronic
     communications, but many of them still prefer face-to-face
encounters.
     A. Although people              B. Despite people
     C. Today people                        D. The fact that people

7.        together in one place, they form a community.
      A. When people who live             B. When people living
     C. Whenever people live              D. Whenever living
people

8.             managed by an independent govemor and board of
directors, the Bank of Canada is owned by the Canadian
government.
      A. And yet B. In spite of it C. Although       D. It is

 9.             pieces of rope are of different thickness, the weaver's
knot can     be used to join them

      A. Two of           B. Whattwo      C. Two such        D. If two

10.           , the seeds of the Kentucky coffee plant.are poisonous.
      A. Until they have been cooked B. Cooking them
92
Chapter Eight Exercises: Test

      C. They have been cooked              D. Cooked until

 11. Natural silk is still highly prized        similiar artificial fabrics.
A. although is available              B. despite there are available
C. in spite of the availability of D. even though an availability of

12. Cattle ranches are found almost                in Utah.
    A. whenever      B. everywhere          C. overall D. somewhere

 13.            through a prism, a beam of white light breaks into all
the colors of the rainbow.
      A. When shines B. It shines C. It is shone   D. When shone

14.           most people think of freezing as a relatively modern
method of food preservation, it is actually one of the oldest.
A. Even B. As though              C. However              D. Although

 15.         large bodies of water never freeze solid is that the sheet
of ice on the surface protects the water below it from the cold air.
A. Because      B. Why do C. The reason that        D. For the reason

16. ---,-_ _ granted by the Patent Office, it becomes the inventor's
property and he or she can keep it, sell it, or licence it to someone
else.
      A. Once a patent is              B. When a patent
      C. A patent, once                D. A patent, whenever it

 17. Owls can hunt in total darkness   their remarkably keen
sense of smell.
     A. since B. because of C. the result          D. that

 18.         most bamboo blooms every year, there are some
species that  flower only two or three times a century.
     A. Whenever    B. That    C. While          D. However

copunro A·6

 Task; Completing structure problems involving in complete noun
clauses.
Directions: Choose the one option out of A,B,C, and D to complete the
sentences with the most appropriate words and clauses..

1.          begin their existence as ice crystals over nost of the
earth seems    likely.

                                                                               93
Chapter Eight Exercises: Test

      A. Raindrops                             B. If raindrops
      C. What if raindrops                     D. That raindrops

 2. Scientists cannot agree on            related to other orders of
insects.
     A. that fleas are                         B. how fleas are
      C. how are fleas                         D. fleas that are

 3. It was 1875            joined the staff of the astronomical
obserbatory at Harvard University.
      A. that Anna Winlock                   B. Anna Winlock, who
      C. as Anna Winlock                     D. Anna Winlock then

4.             is a narrow stop od woods along a stream in an open
     .grassland.
      A, Ecologists use the term "gallery forest"
      B. What do ecologists call a "gallery forest"
      C. "Gallary forest" is the term ecologists use
      D. What ecologists call a "gallary forest"

 5.           developed so rapidly in Alahama primarily because of its
rich natuarl resources.
     A. That heavy industry                 B. Heavy industry
     C. Heavy industry that was             D. When heavy industry

6. _ _...,.,- so incredible is that these insects successfully migrate to
places   that they have never even seen.
     A. That makes the monarch butterflies' migration
     B. The migration of the monarch butterflies is
     C. What makes the monarch butterflies' migration
     D. The migration of the monarch butterflies, which is

7.   Art critics do not all agree on what         a painting great.
     A. qualities make                    B. are the qualities for making
     C. qualities to make                 D. do the qualities that make

8.   In order to grow vegetables properly, gardeners must know

      A. what the requirements TOr each vegetable are
      B. that the requirements for each vegetable
      C. what are each vegetable's requirements
      D. that is required by each vegetable

9. When         is not known.
    A. was the wheel invented            B. the invention of the wheel
94
Chapter Eight Exercises: Test

      C. inventing the wheel              D. the wheel was invented

 10. For many years people have wondered                   exists
elsewhere in the universe.
     A. that life                  B. life which
     C, whether life               D. life as it

 11.            of all modern domestic poultry is the red jungle fowl is
widely     believed.
      A. The ancestor                  B. The ancestor is
      C. How the ancestor              D. That the ancestor

 12.            the right side of a person's brain is dominant, that
person is left-    handed.
     A. That                               B. If
     C. Which                              D. For


copanro      A·7

Task; Completing structure problems involving word order.
Directions: Choose the one option-A,B,C, or D-that correctly
completes the sentence, then mark the appropriate blank.

1. Hills known as land islands, or salt domes, are             Louisiana's
    marshlands.
    A. extremely interesting features of
     B. of extremely interesting features
     C. interesting extremely features of
     D. extremely interesting of features

2.          of chamber music is the string quartet.
      A. The famous most from         B. The most famous form
      C. The form most famous         D. Most the form famous

 3. Not until the seventeenth century          to measure the
speed of light.
    A. did anyone even attempt        B. anyone did even attempt
     C. did anyone attempt even       D. did even attempt anyone

4.    Alfalfa is       for livestock.
      A. a primarily grown crop           B. grown primarily a crop
      C. a crop grown primarily           D. a grown crop primarily


                                                                        95
Chapter Eight Exercises: Test

5. The Franklin stove, which became common in the 1790's, burned
wood        _ _ an open fireeplace.
     A. efficiently more than much   B. much more efficiently than
     C. much more than efficiently   D. more efficiently much than

6. Reinforced concrete is concrete that is strengthened by metal
bars.--:-__
      A. in it that are embedded B. embedded that are in it
       C. are that it embedded in D. that are embedded in it

7. The type of clothing people wear tells others a lot about       _
    A. who they are              B. are they who
    C. they are who              D. who are they

8 Most southern states had set up primary school systems by the
late eighteenth century, but only in New England          and open
to all students.
       A. primary schools were free    B. were primaary schools free
       C. free were primary schools    D. were free primary schools

9. Fungi,       , do not produce chlorophyll.
    A. as such mushrooms                  B. mushrooms as such
    C. such as mushrooms                  D. mushrooms such as

10. Seldom --:---::-_ more than 20 minutes a night.
    A. sleep giraffes                B. do giraffes sleep
    C. giraffes do sleep             D. giraffes sleep

11.            of the early years of space exploration was the
discovery of the Van Allen radiation belt in 1958.

A. Perhaps the greatest triumph
B. The triumph perhaps greatest
C. The greatest perhaps triumph
D.The triumph greatest perhaps
12. Today            major new products without conducting elaborate
market research.
     A. corporations hardly introduce ever
     B. hardly ever corporations introduce
     C. hardly ever introduce corporations
     D. corporations hardly ever introduce

13. Across the Chesapeake Bay from the rest of the state    _
whose farms produce beans, tomatoes, and garden vegetables.
    A. there lies Maryland's Eastern Shore
96
Chapter Eight Exercises: Test

       B. lies Maryland's Eastern Shore
       C. Maryland's Eastern Shore lies there
       D. Maryland's Eastern Shore lies

14. Acidophilus bacteria are ~_ _ in an acid medium.
    A. those that grow best        B. those grow best that
    C, that those grow best        D. grow best those that

15,         of great apes; the gibbon is the smallest.
       A. Four of the types       B, The four of types
       C, Four types of the       D, Of the four types

16, It is difficult    through swamps because of tangled roots
and shallow waterways.
     A. to navigate even for small boats
     B. for even small boats to navigate
     C. Even small boats for to navigate
     D. even to navigate for small boats

 17.    A lodestone is -,,-     _
A. an occurring naturally magnet B. a magnet naturally occurring
C. naturally a magnet occurring D. a naturally occurring magnet

 18, So complicated              that consumers who use a product are
seldom      aware of where all its components come from.
     A. today trade is international     B. today international trade is
     C. is international trade today     D, international trade is today

19. The snow bunting is        winter birds in Canada.
    A. one most of the common       B. the most common one of
     C. one of the most common      D, the one of most common

20,    Nashville has         the capital of country music,
       A. as long been known             B, been known long as
       C. long been known as             D. long as been known


copanro A- 8

Task: Completing structure problems involving infinitive and gerund
     phrases,
     Directions: Choose the one option out of the A,B,C, and D to
complete the sentences with the most appropriate ones,


                                                                      97
Chapter Eight Exercises: Test

1.             for a career in dance generally begins at an early age.
      A. People train                    2. That people train
      C. If training                     4. Training

2.   A baby's first teeth       are generally the lower incisors.
      A. appearance                    B.appear
      C. to appear                     D. in appearing

 3. A climbing helmet           protection for a rocket-climbers head
from falling rocks and other hazards.
     A. to provide                      B. provides
     C. providing                       D. that provides

4. Power tools require careful handling          injuries.
    A. by avoiding                     B. they avoid
    C. to avoid                        D. that avoid

 5. An electromagnet is created              electrical current through
a coil of    wire
       A. by passing                     B. passes by
       C. to be passed                   D. passed

6.           at home requires only three types of chemicals, several
pieces of simple equipment, and running water.

      A. For the development of film.          B. To develop film
      C. When film is developed                D. In developing film

 7. The purpose of cost accounting is             involved in producing
and selling a good or service
    A. as a determination of its costs         B. the costs determined
    C. that determines the costs               D. to determine the costs

8.            was one of the most difficult tasks pioneers faced on
their joumeys west.
      A. Crossing rivers                      B. While crossing rivers
      C. Rivers being crossed                 D. By crossing rivers

9. Energy can be defined as the ability :--:----:__
    A. do working                      B. to do work
    C. doing work                      D. work to be done

10. The process of _-_ _ by hand has changed little since the
fifteenth century.
       A. to bind books                 B.binding books
98
Chapter Eight Exercises: Test

      C. books are bound                     D. bound books
11. A crescent wrench has adjustable jaws for          a nut, bolt, or
pipe.
      A. to grip B. they grip          C. gripping D. gripped
12. Compressed air is         air brakes, pneumatic tools, and other
      machinery.

      A. used to powering               B. to use powering
      C. used to power                  D. in use by powering
13. Some people believe that the crystals of certain minerals         _
curative powers.
      A. have      B. having      C. that have           D. to have
14. The narrow blades of speed skates allow _ _ speeds of up to
30 miles per hour.
      A. for skaters maintaining
      B. skaters to maintain
      C. skaters maintain
      D. maintenance by skaters
15. The first library _ _ in the Nebraska Territory was built in Fort
Atkinson in 1870.
      A. to be established              B. was established
      C. could establish                D. to establish

copanro A-9

    Task: Completing structure problems involving paralleism.
    Directions: Choose the one option out of the A,B,C, and D to
complete the sentence with the most appropriate one.

1. Insects provide many beneficial services, such as        _
breaking down deadwood, and pollinating plants.
     A. they condition soils                B. to condition soil
     C. conditioning the soil               D. soil conditioned

2. Frozen orange juice must be packed,                and stored when
the fruit   is ripe.
      A. be frozen                            B. must be frozen
      C. frozen                               D. it must be frozen

3. The Sioux language is spoken not only              Sioux but also by
the Crow and Osage tribes.
    A. by the                                 B. the
    C. do the                                 D. and the


                                                                      99
Chapter Eight Exercises: Test

4. In 1900 electrically powered cars were more popular than
gaseline· powered cars because they were quiet, opreated smoothly,
and ~--,-_
     A. handled easily                          B. ease of handling
     C. handling easily                         D. easy to handle

 5. Roger Williams was a clergyman,              the colony of Rhode
Island,    and an outspoken advocate of religious and political
freedom.
      A. founded                            B. the founder of
      C. was the founder of                 D. he founded

6. Paint can be applied to a surface with rollers, _ _ , or spray
guns.
      A. brushes                             B. brushes can be used
      C. with brushes                        D. by brush

7, The use of labor-saving devices in homes,            , and in
factories added      to the amount of leisure time people had.
      A. at office                          B. used in offices
      C. offices                            D. in offices

8. A dulcimer can be played by either striking its strings with a
hammer or.,..-,.--,.,.._'
    A. to pluck them with the fingers
    B. fingers are used to pluck them
    C. they are plucked with the fingers
    D. plucking them with the fingers

9. Throughout history, trade routes have increased contact between
people,             , and greatly affected the growth of civilzation.
     A. have resulted in an exchange of ideas
     B. an exchange of ideas has resulted
     C. resulted in an exchange of ideas
     D. resulting in an exchange of ideas

10. Walt Disney made many technical advances in the use of sound,
color,     and         in animated films.
       A. photographing                   B. using photography
       C. photography                     D. use of photographs

11. Artist Paul Kane traveled throughout Northwest Canada on foot,
by canoe, and              to sketch Native Canadians going about-
their ordinary lives.
      A. on horseback                        B. riding a horse
100
Chapter Eight Exercises: Test

      B. was on horseback                     D. by a horse




12. Barbara Jordan was the first woman in the South to win an
election to the House of Representatives,             as
Congresswoman from Texas         from 1973 to 1979.
      A to serve                            B. served
      C. serving                            D. has served

13. Photographers' choice of a camera depends on what kind of
pictures they
      want to take, how much control they want over exposure, and

      they want to spend.
      A the amount of money              B. what money
      C. how much money                  D. so much money that

14.   Atlanta is the commercial, financial, and          of Georgia.
      A center of administration                B. administrative center
      C. center for administering               D. administrating center

15. Even after the Revolutionary War, American importers obtained
    merchandise from Britain because British merchants understood
    American tastes, offered attractive prices, and -:-__
    A easy credit was provided         B. because of easy credit
    C. easy credit                     D. provided easy credit


CopMnro A ·10

Task: Completing structure problems involving prepositional phrases.
Directions: Choose the one option out of of A,B,C, and D, correctly
completing the sentence.

  1.             seed of a flowering plant is covered by a dense
. protective coat.
       AGn each         B. Each     C. Each of        D. That each

2. Dynamite is ordinarily detoned             called a blasting cap.
    A a device is used                        B. that a device
    C. with a device                          D. the use of a device


                                                                       101
Chapter Eight Exercises: Test

3.              1900 there were some 300 bicycle factories in the
United States
     and they produced over a million bicycles.
     A. In       B. Because in C. It was in         D. That in
4. A thick layer of fat called blubber keeps whales warm even        _
coldest water.
     A. although the B. in the C. the               D. of the

5.              the United States, the general movement of air masses
is from west to east.
      A. Across        B. To cross      C. They cross D.lt's across
6. The bark of a tree thickness             _
      A. with age      B. it gets older C. as older D. by age

7. A substance that is harmless to a person who has no allergies can
cause mild to serious reactions in a person      allergies.
     A. has      B. which having C. can have     D. with

8. In 1868 a number of national unions formed the American
Federation of    Labor             ..,.-
     A. Samuel Gompers was its leader
     B. under the leadership of Samuel Gompers
     C. which, under Samuel Gompers' leadership
     D. Samuel Gompers led it
9. Harmonicas. autoharps, and kazoos              folk intruments.
     A. are examples                           B. for example
     C, are examples of                        D. as examples of
10.            charming shops and restaurants, Old Town is the most
     picturesque section of Albuquerque.
     A. With its B. Its        C. Because its D. For its
11.             such as banking and travel, in which computers are not
a convenience but a necessity.
     A. Where some industries,           B. In some industries
     C. Some industries                  D.There are some industries,
12. One of the oldest large suspension bridges still            today is
the
     George Washington Bridge between New York City and Fort
Lee, New Jersey
     A. uses            B. is used       C. the use of      D in use

cepanro A -11

Task: Completing structure problems involving misplaced modifiers.


102
Chapter Eight Exercises: Test

Directions: Choose the one option out of A,B,C, and D and complete
the sentence correctly.

1. Fearing economic hardship, _-:-----:---:-:~:--':-:-:'"-;--
      A. many New Englanders emigrated to the Midwest in the 1820s
      B. emigration from New England to the Midwest took place in
the 1820s
      C. it was in the 1820s that many New Englanders emigrated to
the Midwest
      D. an emigration took place in the 1820s from New England to
the Midwest.
2. Rich and distinctive in flavor:::-:    _
      A. there is in the United States a very important nut crop, the
pecan
      B. the most important nut crop in the Unites States, the pecan
      C. farmers in the United States raise pecans, a very important
nut crop
      D. pecans are the most important nut crop in the United States
3. Orbiting from 2.7 to 5.6 billion miles from the sun ",=",_....,...-....,.
      A. the astronomer Clyde Tombaugh discovered Pluto in 1930
      B. Pluto was discovered by the astronomer Clyde Tombaugh in
1930
      C. it was in1930 that the astronomer Clyde Tombaugh
discovered Pluto
      D. the discovery of Pluto was made by Clyde Tombaugh in 1930
4. A popular instrument --:-_.,....-_
      A. only a limited role has been available to the accordion in
classical music

     B. there is only a limited role for the accordion in popular music
     C. classical music provides only a limited role for the accordion
     D. the accordion has played only a limited role in classical music
5. Unlike most birds,
     A. the heads an-d-:--nec--:-k-s of vultures lack feathers
     B. feathers are not found on the heads and necks of vultures
     C. vultures do not have feathers on their heads and necks
     D. there are no feathers on vultures' heads and necks
6. Widely reproduced in magazines and books, ..,...-_----:--:-:_'
     A. Ansel Adams depicted the Western wilderness in his
photographs
     B. the Western wilderness was depicted in the photographs of
Ansel Adams
     C. Ansel Adams' photographs depicted the Western wilderness
     D. it was through his photographs that Ansel Adams depicted
the WesterQ. wilderness
                                                                         103
Chapter Eight Exercises: Test

7. Smaller and flatter than an orange, .-:-:-_---._
      A. a tangerine is easy to peel and its sections separate readily
      B. the peel of a tangerine is easily removed and its sections are
raedily separated
      C. it's easy to peel a tangerine and to separate its sections
      D. to peel a tangerine is easy, and its sections can be readily
separated
  8. Like the federal government,~--:----:~
      A. taxation provides most of the funds for state and local
governments as well
      B. state and local governments obtain most of thier funds
through taxation
      C. through taxation is how state and local governments obtain
most of their funds
      D. funds are provided from taxation for state and local
governments
9. Originally settled by Polyesians around 700 AD,             _
      A. Hawaii received its first European visitor in 1778, when
Captain James Cook landed there
      B. Hawaii's first European visitor, Captain James Cook, landed
there in 1778
      C. in 1778 the first European, Capatain James Cook, visited
Hawaii
      D. the first European to visit Hawaii was Captain James Cook,
landing there in 1778
10. Unlike most modernist poets,             based on ordinary speech.
      A. Robert Frost's poems were
      B. the works of Robert Frost were
      C. Robert Frost wrote poems that were
      D. the poetry written by Robert Frost was
11. Named for its founder,            in Ithaca, New York.
      A. in 1.865 Ezra Cornell established Cornell University
      B. Cornell University was established in 1865 by Ezra Cornell
      C. it was 1865 that Cornell University was established by Ezra
Cornell
      D. Ezra Cornell established Cornell University in 1865

12. While living in New Orleans,          the Creole people of
Louisiana.
      A. a book of folklore, Bayou Folk, was written by Kate Chapin
about
      B. Bayou Folk, a book of folklore, was written by Kate Chapin
about


104
Chapter Eight Exercises: Test

       C. the subject of Kate Chapin's book Bayou Folk was the
folklore of
       D. Kate Chapin wrote Bayou Folk, a book about the folklore of




copanro 6

,QapaaXb copanrur TyxaH TyxaHH 6yr31., AaanraBapblH Aaryy axannax
Hb TaHbl
X3n 3yHH. TyxaHn6an. eryyn63p 3YHH TyxaH M3Afl3r oanrouur
6aTaraxblH 33p3r1.33r33p eryyn63p 6YPMHH WMH3 yr X3nn3rMHr caHH
rons 6101'·lr33c caarap HRrTnaH Y33>K, copanrur ryH1.33X X3P3rT3H.
copanrur a>KMnnaCHbI napaa eryyn63p TyC 6ypMHr 6yr31.. repen,
eryyn63p ,Q3X yrMHH A3C Aapaa, eryyn63pMHH rMwYV,QMHH 6aHpnanblH
XYBb,Q AaXMH ryuraaa Y33>K 3X x3nHMHx33
X3B X3M>K33HMH Aaryy opayynax Hb TaHbl aHrnM x3nHMH M3Afl3r
op-yynax yp Ya,QBapT TyC 60nHo.

CopMnro 6·1

 Choose the correct answer for the blank. When you choose the
correct answer
pay attention to the use of the verbals.

1. One of the professor's great attributes is             '
     A. when he gives lectures
     B. how in the manner that he lectures
     C. the wat to give lectures
     B. his ability to lecture
2. To get an education, _-:-         _
     A. one must work hard
     B. working hard is one of the most requirements
     C. requirement is needed to work hard
     D. working hard is needed
3. John said that he had run in order        _
     A. that he catch the bus            B. that he can catch the bus
     C. to catch the bus                 D. to the bus he could catch
4. "lVhy was the official meeting called?" "Yes        new officers."

                                                                    105
Chapter Eight Exercises: Test

      A Select                          B. Selecting
      C. To select                      D. For selecting
5. "Where did he go?" " He went to another store.--,-       _
      A to buy slacks            B. for buying slacks
      C. buy slacks              D. buying slacks
6. "I'll help you whenever you need me." "Good. I'd like __ me
tomorrow."
      A you helping              B. that you will help
      C. you to help             D. that you help


7. "The photographer thought he was supposed to arrive after the
quests."
      " But I meant           early."
      A for his coming                    B. him coming
      C. for him to come                  D. he would come
8. " My baby has an infection.."" Did the doctor find it difficult _ _?"
      A. in treating                      B. treating
      C. for treating          ..         D. to treat
9. "What's made Ruth so upset?" "_ _ three tickets to the folk
music       concert."
      A. Lost                       B. Losing
      C. Because of losing          D. Since she lost
10. "What will Andrew be doing in the fall?" "               mathematics
at a private boys' school."
      A. To teach                         B. Teaching
      C. will be teaching                 D. will teach
11. The company manager may enable the men who tend the
machines_ _
      a large panorama of possibilties.
      A to see B. see               C. Seeing          D.seen
12. When they met, Leonardo and his enemy were fighting                  >
      A. killed each other                B. killing each other
      C. to be killed each other          D. to kill each other
13. The president of a company should know                 -
      A. to be firm                       B. the way of firmness
      C. to have firmness                 D. how to be firm
14. "Why were you so late for work today?" "               to the office
was very slow this moming because of the traffic."
      A Driving                     B. To drive
      C. I drove                    D. That II drove
15.              time and labor, cartoonist generally draw the hands of
their characters with only three fingers and a thumb.
      A Saved                       B. Saves
      C. To save                    D. The saving
106
Chapter Eight Exercises: Test

16. Lincoln, Nebraska, is an important manufacturing, insurance, and

      A. shipping of grain           B. to ship grain
      C. grain was shipped           D. grain-shipping
17. There is no            what may happen.
      A. know                        B. knowing
      C. to know                     D. known
18. "How did you spend the night?" "We had a very hard time
_ _.,-some of the problem."
      A. discussing                  B. to discuss
      C. of discussing               D. on discussing
19. With            her do this, she will have no difficulty persuading
them to     accept her plan.
      A. I help                      B. my helping
      C. me helping                  D. mine helping
20. "What are you going to do tomorrow?" "We are going _ _
tomorrow."
      A. to climb                    B. climbing
      C. climbed                     D. climb
21. She must be looking forward as much to his return as he himself
is to          her.
      A. see                         B. have been
      C. seeing                      D. having seen
22. She __ help thinking that she had seen him somewhere
before.
      A. can not                     B. could not
      C. must not                           D. might not
23. He has lots of books,             that he is still young.
      A. considering                 B. considered
      C. being considered                   D. our considering
24. "What would you like for you birthday?" "I'd like the __ works
of Maugham."
      A. collected                          B. collection
      B. collecting                         D. collect
25. The peart is the only gem              by a living creature.
      A. creating                           B. to create
      C. is created                         D. created
26.             to steel, chrominum increases the metal's hardness.
      A. Added                       B. In addition
      B. Adding                      D. Adds
27. Commercial banks make most of their income from interest
_ _.,- on loans and investments in stocks and bonds.
      A. earn                        B. earned
      C. to earn                     D. was earned

                                                                     107
Chapter Eight Exercises: Test

28. "What did they think about the plan?" "Everyone was so happy
and          about it."
       A. excite                  B. exciting
       C. excited                 D: excites
29. "How do you like coffee?" "llike coffee       _
       A. that is sweetened       B. sweetened
       C. being sweetened         D. to be sweetened
30. "How did you learn to drive?" " _ _ strict obedience to my
tutor."
       A. Giving                  B. Give
       C. to give                 D. By giving

31. After taking his examination,          _
       A. the book was read by him
       B. the book made him happy to read it.
       C. he wanted to read the book
       D. the reading of the book gave him some pleasure
32. Anesthetics are used            insensitivity to pain during surgical
       operations.
       A. the cause                      B. to cause         '
       C. cause of                              D. causing J
33. Ganga went to the nicest store in the city _ _ presents for her
children.
       A. for to get                            B. for getting
       C. to get                         D. to getting
34. "Are you ready now?" "Yes, Let's hurry _ _ a seat near the
front."
       A. so to get                             B. for getting
       C. get                                   D. toget
35. At last we found the exhausted animal lying there,            _
       A. sick                           B. to be sick
       C. in sick                        D. to be sickening

Written Expression

Directions: The four underlined parts of the sentence are marked
A,B,C and D.
Identify the one underlined word or phrase that must be changed in
order for the sentence to be correct.

36. Twenty to thirty year after a mature forest is cleaned away, a nearly
                    A B C
impenetrate thicket of trees and shrubs develops.
                                            D
37. The first natural in world, Bogdo Uul Reserve Park, was established     in
108
Chapter Eight Exercises: Test

        A                 B                             C             D
1772.
38. Becauseit doeshavea blood~, the comeatakestheiroxygen direcUy
        A B C                                                       D
from the air.
39. Magnificent mountains and coastal scenery 12 British Columbia's chief
                       ABC                                            D
      tourist attrations.

40. Scientists at universities are often more involved in theoritical research
                         ABC
      than in practically research.
                     D
41. Nylon, a synthetic done from a combination of water, air and a by-product
of coal, was first introduced in 1938.

42.0mithology, the study of birds, is one of the major scientific fields in which
                                                      A              B
      amateur ~ a role in accumulating, researching, and publish data.
                 C                                                 D
43. Animation is a technigue for creativity the illusion of life in inanimate
things.
                         ABC                                                  D
44. On December 7, 1787, Delaware became a first state to ratify the
      A B C                                                          D
      Constitution.
45. Nutritionists believe what diet affects how one feels physically and
         ABC                                                    D
      emotionally.
46. Stars in our universe vary in temperature, color, bright, size and mass.
      A B C                                                                D

47. Ice is less denser than the liquid from which 11 is formed.
                  ABC                             D
48. The 1983 Nobel Prize in Medicine was awarded to Barbara McClintock
for
                                               A
her experiments with maize and her discoveries regardless the nature of
      B C D
      DNA.
49. ill 1866 to 1883, the bison population in North American was reduced
     A                                                                   B
     from an estimated 13 million to a few hundred.
                C                           D
50. Most of the damage property attributed ill the San Francisco earthquake
                         A                     B
      Qf 1906 resulted from the fire that followed.
      C                                    D

                                                                             109
Chapter Eight Exercises: Test


copanro 6-2

 The test is related with a complex sentence or clauses: a main clause
and a subordinate clause. So, before doing the test you may review
the clauses.
Choose the correct one out of A,B,C, and 0 to complete the sentences
correctly.

1.          some mammals came to live in the sea is not known.
    --p:.-Which only brown thrashers               B. Since
       C. Although                                 D. How
 2.              all behavior is learned behavior is a basic assumption of
social scientists.
       A. Nearly                            B. That nearly
       C. It is nearly                             D. When nearly
3. George _ _ he could improve his test scores, but he did not
have enough time to study.
       A. knew to                           B. knew how
       C. knew how that                            D. knew how to
4. The people at the party were worried about Janet Because no one
was aware               she had gone.
       A. where that                        B. of where
       C. of the place where                D. the place
5.              progress helps to relieve scarcities is a fact accepted by
economists.
       A. Technological                            B. That technological
       C. Although technological                   D. Ther is technological
6. "Do you remember where _ _ my watch?"
       A. had I put                                B. had put I
       C. I had put                                D. put I
7. I wonder how many years ago _ _.
       A. did your father retire                   B. your father retired
       C. has your father retired                  D. your father has retired
8. "Suie has't written us for such a long time."
       A. that she happen                          B. happened
       C. to happen                                D. having happened
9. There are very few areas in the world                be grown
successfully.
       A. where apricots can                       B. apricots can
       C. apricots that can                        D. where can apricots
10. Caves and hollow trees are not the only places ......,.,._-,-_
       A. where do bats live                       B. bats live where
       C. where bats live                          D. live where bats

110
Chapter Eight Exercises: Test

11. "May I have the loan?" " _ _ you offer good security."
      A. But                                            B. Unless
      C. Provided                                        D. But for
12. ---:----=::--" heat is produced.
      A. The mixing together of certain chemicals
      B. Whenever certian chemicals are mixed together
      C. Certain chemicals mixed together
      D. That certain chemicals are mixed together
13. However much                     , it will be worth it.
      A. does the watch cost                             B. costs the watch
      C. the watch will cost                             D. the watch costs
14. Gorillas are quiet animals,                  they are able'to make about
twenty         different sounds.
      A. how                                     B. in spite of
      C. because of                              D. even though
15.              that the formation of the sun, the planets, and other stars
began          with condensation of an interstellar gas cloud.
      A. Believing                                       B. To believe
      C. The belief                                      D. It is believed
16.                sandstone is broken apart, it is usually the cementing
material       that fractures.
      A. a                        B. in a        C. It is a           D. When a
17. _ _ is indispensible to plant and animal life.
      A. Nitrogen                                B. It is nitrogen
       C. That nitrogen                          D. Although nitrogen
18. As soon as the gate opened,                      -:--_
      A. we found that the two men emerged
       B. the two men had emerged
       C. here the two men emerged
       D. the two men emerged
19. The first doll             say "mama" was invented in 1830.
      A. that it could                           B. could it
       C. it could                               D. that could
20.              relatively costly, the diesel engine is highly efficient and
needs          servicing infrequently.
      A. Even                                    B. It is
       C. Even though                            D. There is
21. We might still catch the train if we -:---:-----:-_
      A. make hurry                              B. haste
       C. make haste                             D. hastily
22. Almost all economists agree                      by trading with one
another.
      A. nations that are gained                         B. nations they gain
       C. gaining nations                                D. that nations gain

                                                                              111
Chapter Eight Exercises: Test

23.               dog was the first animals to be domesticated is
generally agreed
       upon by authorities in the field.
       A. Until the                          B. It was the
       C. The                                D. That the
24.               kinds of dinosaurs were dying out all through the Age of
Reptiles is true.
       A. Some                               B. When some
       C. Some were                          D. That some
25. On the side of the hill, there is a             which was once the
park.
       A. deep hole in the ground            B. hole deep in ground
       C. deep hole in the ground            D. deep in the ground hole
26.                within the algae began very early is evident.
       A. Evolution that was                 B. That evolution
       C. There was evolution                D. It is estimated that
27. All of the plants now raised on farms have been developed from
plants                   wild.
       A. once they grow                            B. they grow once
       C. that once grow                            D. once grow
28. Seeds usually germinate                   the temperature is favorable.
       A. If         B. whereas C. as a result D. in consequences
29. The knee is the joint             the thigh bone meets the large bone
of the        lower leg.
       A. when             B. where          C. why        D. which
30. The costs of distribution and sales make up a large part of the
prices that _-:-:--:--
       A. are paid for all products          B. all products are paid for
       C. for which all products are paid D. for all products paid
31. Neon is said to be inert _ _ does not react easily with other
substances.
       A. because of it                      B. because it
       C. it is because           .          D. is because it
32. ___" glasses can correct most sight defects in healthy eyes.
       A. When well fitted                          B. Well fitted when
       C. Well fitted if                     D. If well fitted when
33.             he has created striking stage sittings for the Martha
Graham dance company, artist Isamu Noguchi is more famous for his
sculpture.
       A. But not                            B. Nevertheless
       C. In spite of                        D. Although
34. A logarithm is ~__ in algebra as an exponent.
       A. known what                         B. known what it is
       C. what is known                      D., what it is known

112
Chapter Eight Exercises: Test

35. Although many colonial scholars consider Joanthan Edwards
important writer,         any more.
      A. though few people read his works
      B. but few people read his works
      C. and his works are not widely read
      D. his works are not widely read
36. The averagetemperature on Mars,the foothplanet from the sun,is about
             A B C
      eighty degrees than colder on Earth.
                             D
37. One of the longest wars in historywere the Hundred Years War, fought
                  A B C
between England and France in the fourteenth and fifteenth centuries.
                                                                  D



38.Any material that l§. attractive by a magnet is by definition magnetic"
                         ABC                                            D
39.What are common known as "Iead"pencils are not lead. but rather a
                   A B C
      mixture of graphite, clay and wax.
                                      D
40. In the last two decades, Bombay and Madras are developed into
centers of
                                                              A
B    C
      the Indian film Industrv.
                       D
41. The abilities to work hard, follow directions, and thinking
indepedently are
                     A                                     B
C
        some of the criteria for success in the work place.
                                D
42. The average cat sleeps sixteen hours for a day, in short intervals
called"

cat naps"
    D
43. The elephant relies more on its sense of smell than for any other
sense.
                            ABC                                     D
44. Some insects hear ultrasonic sounds more than two octaves than
higher
                                                                       113
Chapter Eight Exercises: Test

                A      B                       C
D
        humans can.
45. To stay warm in cold weather, cold-blooded animals must expose
itself to g
            A
B
      source of warmth such as direct sunlight.
          C                         D
46. In the wild, tea plants become trees of approximately thirty feet
high.
        A B C                                                         D
47. Accounting is described as art of classifying, recording, and
reporting
                                  A                    B
      significant financial events.
           C                    D
48. The development of the watch depended upon the invent of the
mainspring.
            A B C D
49. Physical fitness activities can lead to an alarming variety of
injuries if
                           A                                        B
     participants push themselves greatly hard.
                         C         D
50. The structure but behavior of many protozoans are amazingly
complex for
                   A              B                       C
     single-celled animals.
                    D
copanro 6-3

This test is related with some Confusing Verbs. When you do the test
pay attention to the Confusing Verbs. Choose the correct one out of
A,B,C, and B.

1. Before discussing the steps in detail, -,--,-__
     A. a general principle should be laid down
     B. I should like to lie down a general principle
     C. I'd like to lay down a general principle
     D. a general principle lies
2. The typewriter _---,,,--..,........,
     A. Was laying on the table, where it had laid all week

114
Chapter Eight Exercises: Test

     B. was lying on the table, where it had laid all week
     C. was lying on the table, where it had been laid all week
     D. was lying on tha table where it had been laid all week
3. "What dis Joe do?" "He                asleep all morning.
     A. lain                                    B. laid
     C. lay                                     D. lying
4. Up till then, these problems had been              for centuries.
     A. laying dormant                          B. laying dormantly
     C. lying dormant                           D. lain dormantly
5. During the period of inflation, the value of money drops as

      A. prices rise                              B. prices arise
      C. the price is rising               D. prices are raised
6. "Prices keep going up". " Yes, but the interest on my savings
account is certainly not              "
      A. risen                             B. raise
      C. on the rise                       D. on the raise
7. "When did the lecture begin?" "When all the students            _
the professor began his lecture.
      A. seated                            B. sit
      B. were seated                       D. seat
8. My uncle says that:_ _-:c--_
      A. my aunt sat her suitcase in corner
      B. he rises at five o'clock every morning
      C. the river had raised two feet during the night
      D. from where he was lying, he could see a small black box
setting on the table.
9. It is said that          _
      A. for six months the factory machinery has laid idle
      B. I finished laying the bricks, but they did not lay evenly
      C. my mother immediately set the kettle on the stove
      D. we must have setr three hours waiting for him
10. Most of the begining students cannot understand what Professor
Majid

     A. talks                          B. says about
     C. says                           D. discusses about
11. "My watch          five o'clock." "It's very late. I must go".
     A. says                           B. tells
     C. speaks                         D. talks
12. "What do you need?" "Could you                me ten dollar?'
     A. borrow                         B. borrowing
     C. lend                           D. lending
13. "What do you need?" "My pen is out of in!c, may I           ?
     A. use yourrs pen                 B. lend yours
                                                                       115
Chapter Eight Exercises: Test

      C. borrow yours                    D. will borrow
14. When you go to the library tomorrow, _ _-.,.-_
      A. please go to check the reference room if Mary were there
      B. please take this book to Mary
      C. please bring the note to the librarian
      D. please bring this book to the librarian in the reference room
15. "Oh, glad to see you, John." "What            you to this big city?
Be here long?
     A. carries                          B. brings
      C. takes                           D. makes
16. "Why do you like that place?" "I _ _in that city."

      A. bome                            B. had bomed
      C. bear                            D. was born -



17. "He has been working very hard recently." "Because the burden
of a big family has been            on his shoulder."
      A. bear                              B. borne
      C. born                              D. bore
18. "What does the judqe want me to do?" "He wants you to,           _
      A. sayan oath                        B. tell on oath
      C. do an oath                        D. take an oath
19. The visiting delegates were urged to ,_:-:---:-:----:-
      A. talk at library                   B. state their open minds
      C. make individual expressions              D. speak freely
20. He             me all about his trip.
      A. said                              B. talked
      C. talks                             D. tells
21. 'What haopenedtothe leader of the mob?"" He was_at down."
      A. hang                              B. hung
      C. hanged                            D. hanging
22. The teacher has              the story many times.
      A. told                              B. stated
      C. said                              D. talked
23. While searching for the wreckage of s unidentified aircraft, the
Coast
               ABC
      Guard encountered severe squalls at sea.
                                       o
24. Although a number of police officers was guarding the priceles
treasures in                                             A
.13

116
Chapter Eight Exercises: Test

       the museum, the director worried that someone would try to
steal them.
                                                             C        D
25. Since it was so difficult for American Indians to negotiate a peace
treaty or
                        A                               B
      declare war in their native language, they used a universal
understood
                               C                            D
      form of sign language.
26. While verbalisation is the most common form of language in
existance,
                                        A
B
      humans make use of many others systems and techniques to
express
                                               C
D
      their thoughts and feelings.
27. The need for a well-rounded education was an idea expoused
by the
                 A B C
      Greeks in time of Socrates.
                      D
28. Writers and media personnel sell theirsetves best by the
impression given
                               ABC
      in their verbal expression.
                            D
29. In the spirit of the naturalist writers, that author's work portrays
man's
          A B C
      struggle for surviving.
                         D
30. Stephen Crane's stOry is .§ clinical portral of man as an animal
trapped by
                 A B C
      the fear and hunger.
              D
31. Their silly, whiny convesation on a child level was meant to create
tension
                                             A                        B
      and heighten Nancy's fears and anxiety.
             C              0

                                                                      117
Chapter Eight Exercises: Test

 32. For a long time, this officials have been known throughout the
country as
                            A               B
C
     politcal bosses and law enforcers.
                                D
33. The development of the watch depended upon the invent of the
               A B C D
     mainspring.
34. The ordeal of the Cherokee Indians, who were forcible moved
from their
            A                                           B
       homeland in the 1830's, is remembered as the "Trail of Tears."
                    C                     D
35. Physical fitness activities can lead to an alarming variety of
injuries if
                           A                                           B
      participants push themselves greatly hard.
                                C         D
36. The structure but behavior of many protozoans are amazingly
complex
                     A               B                         C
      for single-celled animals.
                            D
37. Alaska's rough climate and terrian divide the state into isolated
regions,                                       A                      B
      making highway maintainance difficulty.
            C                               D

38. For hundreds of years, sailors relied on echoes to warn them of
another
                                        A               B
C
      ships, icebergs, or cliffs in fQggy weather.
                                      D
39. Although he is employed in the scientific and technical fields, the
metric
                  A                                     B
     system is not generally utilized in the United States.
          C                        D
40. Nora hardly never misses an opportunity to.l2ill.v in the tennis
tournaments.
                  A           B                   C D

118
Chapter Eight Exercises: Test

41. Air pollution, together with littering, are causing many problems in
oor                                 ABC
D
      large, industrial cities today.
42. Because of the severe snow storm and the road blocks, the air
force
            A                                                   B
      dropped food and medical supplies close the city.
                e                                D
43. Hummingbirds are the only birds capable to fly backward as well
as
                                 ABC
     forward, up, and down.
          D
44. The news of the president's treaty negotiations with the foreign
government
      A
     were recieved with mixed emotions by the citizens of both
governments.
        B C D
45. Angie's bilingual ablitity and previous experience were the qualities
that
                                                        A
      which helped her get the job over all the other candidates.
        B e D
46. Joel giving up smoking has caused him to gain weight and
become irritable
      ABC
D
      with his acquaintances.
47. They asked me what did happen last night, but I was unable to tell
them.
                              A        B                   C
D
48. The test administrator ordered we not to open our books until he
told us to
           A                        B        C
D
      do so.
49. Our new neighbors had been living in Arizona since ten years
before
         A                     B                    C
D
      moving to their present house.

                                                                      119
Chapter Eight Exercises: Test

50. I would of attended the meeting of the planning committee last
week, but!
          A                                  B
      had to deliver a speech at a convention.
              C                        D
ccpanro 6-4

Structure and Written Expression

Choose the one word or phrase that best completes the sentence.
1. The cyclist              he crossed the main street.
      A. looked with caution after
      B. had looked acutiously before
      C. was looked cautous when
      D. looks cautious when
2. Here               notebook and report that I promised you last week.
      A. is the B.are the C. was the D. has been a
3. Neither Jane nor her brother                a consent form for
tomorrow's         field trip.
      A. need      B. needs C. is needing               D. has need
4. Cuba is            sugar-growing areas in the world.
      A. one of the larger                  B. one of largest
      C. one of the largest          D. largest
5. The skiers would rather              through the mountains than go by
bus.
      A. to travel on train                 B. traveled by train
      C. travel by train             D. traveling by the train
6. That magnificent               temple was constructed by the Chinese.
      A. eight-centuries-old         B. eight-century's-old
      C. old-eight-centuries         D. eight-century-old
7. There were two small rooms in the beach house,                  served
as a
      kitchen.
      A. the smaller of which               B. the smallest of which
      C. the smaller of them         D. smallest of that
8. Pioneer men and women endured terrible hardships, and                _
      A. so do their children         B. neither did the children
      C. also the childs                    D. so did their children
9. Last year, Matt earned                 his brother, who has a better
position.
      A. twice as much as             B. twice more than
      C. twice as many as             D. twice as more as
10.                  ,he would have been able to pass the exam.

120
Chapter Eight Exercises: Test

      A. If he studied more B. if he were sdudying to a greater degree
       C. Sduying more         D. Had he studied more
11. Mr.Duncan does not know               the lawn mower after they had
finished using it.
      A. where did they put          B. where they did put
       C. where they put                   D. where to put
12. The facilities of the older hospital           ---,-;-
      A. is as good or better than the new hospital
      B. are as good or better that the new hospital
      C. are as good or better than the new hospital
      D. are as good as or better than those of the new hospital
13.            pandas eat bamboo almost exclusively, they are also
carnivorous.
      A. Not only              B. Untill   C. As soon as D. Although
14. Although _ _a country illegally is risky, the alien who finds
work may believe the risk worthwhile.
      A. when entering                           B. he enters
      C. entering                                D.havin g entered
15. The Andean condor glides on air currents and doesn't flap its
wings__            it must do so to reach updrafts.
      A. becuase               B. until    C. that          D. as if

 16. __ sighting an approaching car, some drivers tend to speed up.
     A. When slowing down instead of
     B. Instead when slowing down at
     C. When instead of slowing down
     D. Instead of slowing down when
17.         to England remain strong, the Channel Islanders are
exempt from most British taxes.
     A. Although their ties        B. Although tied
     C. Before their ties                D. Tied
18. _ _ , the travlers found that their flight had been canceled
because of the severe snowstorm.
     A. That they arrived at the airport
     B. As soon as arriving at the airport
     C. At the airport
     D. They arrived at the airport

19. When competing in a demolition derby, _ _ until their ccars are
     demolished.
     A. that drivers continue
     B. drivers must continue
     C. drivers continuing
     D. although drivers must continue

                                                                    121
Chapter Eight Exercises: Test

20. _ _ governments point with pride to icreasing mechanisation in
agriculture, human and animal power still produces a significant
portion of the world's food.
      A. Since B. Because           C. So that         D. While

21.          , tobacco farmers had not yet felt its effect.
      A. Though a campaign against smoking
     B. That there was a campaign against smoking
     C. Even though there was a campaign against smoking
     D. There was a campaign against smoking
22. There were few settlements along the North Caroline coast_ _
     many problems for seafarers.
     A. because the offshore barrier posed
     B. before posing the offshore barrier
     C. while posing the offshore barrier
     D. that the offshore barrier had posed
23.        since the death of her father.
     A. The ancestral home of my mother abandoned
     B. My mother's ancestral home standing abandoned
     C. My mother's ancestral home has stood abandoned
     D. My mother's ancestral home which has stood abandoned

24.            , the owner and buyer finally agreed on a price for the
house.
      A. They had been bargaining for several weeks
      B. After bargaining for several weeks
      C. After several weeks they began bargaining
      D. As if bargaining for several weeks
25. _ _ of the tranquilizer, the scientist put a tag on its ear and
recorded details about the animal.
      A. While under the effect the deer
      B. While being under the effect the deer
      C. While the deer was under the effect
      D. While the deer under the effect
 26. He had been cutting human hairs for two years before he came to
                              ABC                                    0
      Darhkan.
27. Linseed oil exposed to airs forms a robbery film.
                     ABC                           0
28. The principal called the teachers' attentions to the fact that, in the
coming
                                       A                  B
C
      year, the school's enrollment would be almost doubled.
                                                    o
122
Chapter Eight Exercises: Test

              e          0
38. Leading scientists are often the kinds of person who have enjoyed
              A B e
      intellectual challenges all their lives.
                                        o
39. Plants of these type grow best in places where there ;s a great
deal of
                   A         B                       e 0
      shade.
40. In spite of their aloofness and independent bearing, the
domisticated cat
         A        B
      has come to depend upon man for food and protection.
              e           0
41. When Rhodesia declared their independence from England few
thought
                                    A                               B
      that the new government would last even a month.
                                         e           0
42. There are almost a million people with Spanish surnames in Los
Angeles
          A
      out of a total population of more than seven millions.
          B e D
43. In the early 1970's total employment in the United States stood at
roughly
                                 A                              B
e
      eighty three millions.
                         o
44. In the 1930's more than two thousands interview of former slaves
were
                          A                      B
      conducted by members of the Federal Writers Project.
              e                 0
45. Both as a hobbyandas a profession, phdographyhasfascinated people
         A                                               B
     for more than hundred years.
             e           0
46. For all their knowledge and years' observations, astronomers have more
        A                                        B
      questions than answers about Halley's or any other comet.
                                    e .~               0

124
Chapter Eight Exercises: Test

47. The tamarind is an attractive tropical tree that may grow as tall as
seventy-
                             A               B        C
     five foot.
             o
48. Though the dust illY heavy upon the floor, it was evident that an
old house,
                         A                                          B
    its windows carelesly boarded gp, must once have been a
magnificent
                                       C          0
      mansion.
49. The Scots have porridge for breakfast; a wedding breakfast was
held in
           ABC                                      0
      her father's house.
50. Prime Minister announced the names of those who were to be
named to
             A            B                              C
o
      the Cabinet.




                                                                        125
English syntax
English syntax

Contenu connexe

Tendances

CLASS VIII SLATE ENGLISH WITH KEY
CLASS VIII SLATE ENGLISH WITH KEY CLASS VIII SLATE ENGLISH WITH KEY
CLASS VIII SLATE ENGLISH WITH KEY Rc Os
 
De hoc sinh gioi lop 10
De hoc sinh gioi lop 10De hoc sinh gioi lop 10
De hoc sinh gioi lop 10Fant Kute
 
Bài tập đọc hiểu luyện thi lớp 10 môn tiếng anh
Bài tập đọc hiểu luyện thi lớp 10 môn tiếng anhBài tập đọc hiểu luyện thi lớp 10 môn tiếng anh
Bài tập đọc hiểu luyện thi lớp 10 môn tiếng anhHọc Tập Long An
 
General education
General educationGeneral education
General educationAlex Acayen
 
Bai giai chi tiet mon anh (chuyen) tuyen sinh lop 10 tphcm 2013
Bai giai chi tiet mon anh (chuyen) tuyen sinh lop 10 tphcm   2013Bai giai chi tiet mon anh (chuyen) tuyen sinh lop 10 tphcm   2013
Bai giai chi tiet mon anh (chuyen) tuyen sinh lop 10 tphcm 2013Tommy Bảo
 
Taller preparatorio-icfes-inglc3a9s-convertido
Taller preparatorio-icfes-inglc3a9s-convertidoTaller preparatorio-icfes-inglc3a9s-convertido
Taller preparatorio-icfes-inglc3a9s-convertidoJuanDavid614
 
Tthudhtad121
Tthudhtad121Tthudhtad121
Tthudhtad121Huynh ICT
 
Q3 extended
Q3   extendedQ3   extended
Q3 extendedsparkly
 
Bcs preliminary
Bcs preliminaryBcs preliminary
Bcs preliminaryItmona
 
UNite Kindom
UNite KindomUNite Kindom
UNite Kindomtracy_su
 
Reading vocabulary
Reading vocabularyReading vocabulary
Reading vocabularyglenda75
 
CLASS V SLATE EVS WITH KEY
CLASS V SLATE EVS WITH KEY CLASS V SLATE EVS WITH KEY
CLASS V SLATE EVS WITH KEY Rc Os
 

Tendances (19)

Ingles (1)
Ingles (1)Ingles (1)
Ingles (1)
 
CLASS VIII SLATE ENGLISH WITH KEY
CLASS VIII SLATE ENGLISH WITH KEY CLASS VIII SLATE ENGLISH WITH KEY
CLASS VIII SLATE ENGLISH WITH KEY
 
Sack race quiz bee mechanics
Sack race quiz bee mechanicsSack race quiz bee mechanics
Sack race quiz bee mechanics
 
De hoc sinh gioi lop 10
De hoc sinh gioi lop 10De hoc sinh gioi lop 10
De hoc sinh gioi lop 10
 
Bài tập đọc hiểu luyện thi lớp 10 môn tiếng anh
Bài tập đọc hiểu luyện thi lớp 10 môn tiếng anhBài tập đọc hiểu luyện thi lớp 10 môn tiếng anh
Bài tập đọc hiểu luyện thi lớp 10 môn tiếng anh
 
General education
General educationGeneral education
General education
 
Bai giai chi tiet mon anh (chuyen) tuyen sinh lop 10 tphcm 2013
Bai giai chi tiet mon anh (chuyen) tuyen sinh lop 10 tphcm   2013Bai giai chi tiet mon anh (chuyen) tuyen sinh lop 10 tphcm   2013
Bai giai chi tiet mon anh (chuyen) tuyen sinh lop 10 tphcm 2013
 
Taller preparatorio-icfes-inglc3a9s-convertido
Taller preparatorio-icfes-inglc3a9s-convertidoTaller preparatorio-icfes-inglc3a9s-convertido
Taller preparatorio-icfes-inglc3a9s-convertido
 
Tthudhtad121
Tthudhtad121Tthudhtad121
Tthudhtad121
 
Q3 extended
Q3   extendedQ3   extended
Q3 extended
 
Gmat 1000 Sc
Gmat 1000 ScGmat 1000 Sc
Gmat 1000 Sc
 
9no english(c)
9no english(c)9no english(c)
9no english(c)
 
TOEFL : Reading
TOEFL : ReadingTOEFL : Reading
TOEFL : Reading
 
Bcs preliminary
Bcs preliminaryBcs preliminary
Bcs preliminary
 
Uk
UkUk
Uk
 
UNite Kindom
UNite KindomUNite Kindom
UNite Kindom
 
Reading vocabulary
Reading vocabularyReading vocabulary
Reading vocabulary
 
De Anh D Ct M174
De Anh D Ct M174De Anh D Ct M174
De Anh D Ct M174
 
CLASS V SLATE EVS WITH KEY
CLASS V SLATE EVS WITH KEY CLASS V SLATE EVS WITH KEY
CLASS V SLATE EVS WITH KEY
 

Similaire à English syntax

.;.......•.ExaminationLesson 2 Revising and Editi.docx
.;.......•.ExaminationLesson 2 Revising and Editi.docx.;.......•.ExaminationLesson 2 Revising and Editi.docx
.;.......•.ExaminationLesson 2 Revising and Editi.docxmercysuttle
 
Đề thi thử THPT 2018 môn Tiếng Anh Trường THPT Ba Đình - Hà Nội lần 1
Đề thi thử THPT 2018 môn Tiếng Anh Trường THPT Ba Đình - Hà Nội lần 1Đề thi thử THPT 2018 môn Tiếng Anh Trường THPT Ba Đình - Hà Nội lần 1
Đề thi thử THPT 2018 môn Tiếng Anh Trường THPT Ba Đình - Hà Nội lần 1VuKirikou
 
Tuyen tap 20 nam de thi olympic tieng anh lop 11
Tuyen tap 20 nam de thi olympic tieng anh lop 11Tuyen tap 20 nam de thi olympic tieng anh lop 11
Tuyen tap 20 nam de thi olympic tieng anh lop 11Nguyen Van Tai
 
DOUBLE TROUBLE GEN QUIZ.pptx
DOUBLE TROUBLE GEN QUIZ.pptxDOUBLE TROUBLE GEN QUIZ.pptx
DOUBLE TROUBLE GEN QUIZ.pptxArul Mani
 
Тест по страноведению США
Тест по страноведению СШАТест по страноведению США
Тест по страноведению СШАIrene_Ermolova
 
OPEN DAY 2023 - Gen-Quiz
OPEN DAY 2023 - Gen-QuizOPEN DAY 2023 - Gen-Quiz
OPEN DAY 2023 - Gen-QuizSJU Quizzers
 
FASH243-Homework-Help-10.pdf
FASH243-Homework-Help-10.pdfFASH243-Homework-Help-10.pdf
FASH243-Homework-Help-10.pdfsoftware1111
 
Sfbay quiz jan2013_withans
Sfbay quiz jan2013_withansSfbay quiz jan2013_withans
Sfbay quiz jan2013_withansArun Simha
 
Burning bridges 5.0 - MELA-ish General Hub Quiz
Burning bridges 5.0 - MELA-ish General Hub QuizBurning bridges 5.0 - MELA-ish General Hub Quiz
Burning bridges 5.0 - MELA-ish General Hub QuizPratyush Pran Sarma
 
Let general education 5
Let general education 5Let general education 5
Let general education 5Alex Acayen
 
Answer Key - Guide to Reading.pdf
Answer Key - Guide to Reading.pdfAnswer Key - Guide to Reading.pdf
Answer Key - Guide to Reading.pdfPradip khatri
 
General Quiz by HeadRush, DA-IICT
General Quiz by HeadRush, DA-IICTGeneral Quiz by HeadRush, DA-IICT
General Quiz by HeadRush, DA-IICTKunal Doshi
 

Similaire à English syntax (20)

English syntax
English syntaxEnglish syntax
English syntax
 
.;.......•.ExaminationLesson 2 Revising and Editi.docx
.;.......•.ExaminationLesson 2 Revising and Editi.docx.;.......•.ExaminationLesson 2 Revising and Editi.docx
.;.......•.ExaminationLesson 2 Revising and Editi.docx
 
Đề thi thử THPT 2018 môn Tiếng Anh Trường THPT Ba Đình - Hà Nội lần 1
Đề thi thử THPT 2018 môn Tiếng Anh Trường THPT Ba Đình - Hà Nội lần 1Đề thi thử THPT 2018 môn Tiếng Anh Trường THPT Ba Đình - Hà Nội lần 1
Đề thi thử THPT 2018 môn Tiếng Anh Trường THPT Ba Đình - Hà Nội lần 1
 
Tuyen tap 20 nam de thi olympic tieng anh lop 11
Tuyen tap 20 nam de thi olympic tieng anh lop 11Tuyen tap 20 nam de thi olympic tieng anh lop 11
Tuyen tap 20 nam de thi olympic tieng anh lop 11
 
KQA Mega-Whats 2015 Prelims
KQA Mega-Whats 2015 PrelimsKQA Mega-Whats 2015 Prelims
KQA Mega-Whats 2015 Prelims
 
MFQT '19 Prelims
MFQT '19 PrelimsMFQT '19 Prelims
MFQT '19 Prelims
 
DOUBLE TROUBLE GEN QUIZ.pptx
DOUBLE TROUBLE GEN QUIZ.pptxDOUBLE TROUBLE GEN QUIZ.pptx
DOUBLE TROUBLE GEN QUIZ.pptx
 
Тест по страноведению США
Тест по страноведению СШАТест по страноведению США
Тест по страноведению США
 
OPEN DAY 2023 - Gen-Quiz
OPEN DAY 2023 - Gen-QuizOPEN DAY 2023 - Gen-Quiz
OPEN DAY 2023 - Gen-Quiz
 
General Quiz
General QuizGeneral Quiz
General Quiz
 
Qc meet
Qc meet Qc meet
Qc meet
 
FASH243-Homework-Help-10.pdf
FASH243-Homework-Help-10.pdfFASH243-Homework-Help-10.pdf
FASH243-Homework-Help-10.pdf
 
Saturday Evening Quiz Club, IIM Ranchi
Saturday Evening Quiz Club, IIM RanchiSaturday Evening Quiz Club, IIM Ranchi
Saturday Evening Quiz Club, IIM Ranchi
 
Sfbay quiz jan2013_withans
Sfbay quiz jan2013_withansSfbay quiz jan2013_withans
Sfbay quiz jan2013_withans
 
Burning bridges 5.0 - MELA-ish General Hub Quiz
Burning bridges 5.0 - MELA-ish General Hub QuizBurning bridges 5.0 - MELA-ish General Hub Quiz
Burning bridges 5.0 - MELA-ish General Hub Quiz
 
General Quiz
General Quiz General Quiz
General Quiz
 
Let general education 5
Let general education 5Let general education 5
Let general education 5
 
Guest Quiz
Guest QuizGuest Quiz
Guest Quiz
 
Answer Key - Guide to Reading.pdf
Answer Key - Guide to Reading.pdfAnswer Key - Guide to Reading.pdf
Answer Key - Guide to Reading.pdf
 
General Quiz by HeadRush, DA-IICT
General Quiz by HeadRush, DA-IICTGeneral Quiz by HeadRush, DA-IICT
General Quiz by HeadRush, DA-IICT
 

Plus de Munkh Orgil

School budgeting 2016
School budgeting 2016School budgeting 2016
School budgeting 2016Munkh Orgil
 
сайдын тушаал-7-төсвийн-ангилал-шинэчлэн-батлах-тухай
сайдын тушаал-7-төсвийн-ангилал-шинэчлэн-батлах-тухайсайдын тушаал-7-төсвийн-ангилал-шинэчлэн-батлах-тухай
сайдын тушаал-7-төсвийн-ангилал-шинэчлэн-батлах-тухайMunkh Orgil
 
шилэн дансны хууль журмын танилцуулга
шилэн дансны хууль журмын танилцуулгашилэн дансны хууль журмын танилцуулга
шилэн дансны хууль журмын танилцуулгаMunkh Orgil
 
Zorilt 2014-2015
Zorilt 2014-2015Zorilt 2014-2015
Zorilt 2014-2015Munkh Orgil
 
Hudaldan avah last-1
Hudaldan avah last-1Hudaldan avah last-1
Hudaldan avah last-1Munkh Orgil
 
Tesviin tuhai-huuli
Tesviin tuhai-huuliTesviin tuhai-huuli
Tesviin tuhai-huuliMunkh Orgil
 
Sanhuu tesviin-zaavar
Sanhuu tesviin-zaavarSanhuu tesviin-zaavar
Sanhuu tesviin-zaavarMunkh Orgil
 
2014 state budget toim dugnelt osf
2014 state budget toim dugnelt osf2014 state budget toim dugnelt osf
2014 state budget toim dugnelt osfMunkh Orgil
 
хүний хөгжил-сангийн-2014-оны-төсвийн-тухай
хүний хөгжил-сангийн-2014-оны-төсвийн-тухайхүний хөгжил-сангийн-2014-оны-төсвийн-тухай
хүний хөгжил-сангийн-2014-оны-төсвийн-тухайMunkh Orgil
 
нийгмийн даатгалын-тухай-хууль
нийгмийн даатгалын-тухай-хуульнийгмийн даатгалын-тухай-хууль
нийгмийн даатгалын-тухай-хуульMunkh Orgil
 

Plus de Munkh Orgil (20)

264 tog
264 tog264 tog
264 tog
 
263 tog
263 tog263 tog
263 tog
 
2017 3631 1
2017 3631 12017 3631 1
2017 3631 1
 
Togtool 242
Togtool 242Togtool 242
Togtool 242
 
School budgeting 2016
School budgeting 2016School budgeting 2016
School budgeting 2016
 
сайдын тушаал-7-төсвийн-ангилал-шинэчлэн-батлах-тухай
сайдын тушаал-7-төсвийн-ангилал-шинэчлэн-батлах-тухайсайдын тушаал-7-төсвийн-ангилал-шинэчлэн-батлах-тухай
сайдын тушаал-7-төсвийн-ангилал-шинэчлэн-батлах-тухай
 
шилэн дансны хууль журмын танилцуулга
шилэн дансны хууль журмын танилцуулгашилэн дансны хууль журмын танилцуулга
шилэн дансны хууль журмын танилцуулга
 
Zorilt 2014-2015
Zorilt 2014-2015Zorilt 2014-2015
Zorilt 2014-2015
 
4
44
4
 
3
33
3
 
2
22
2
 
5
55
5
 
Hudaldan avah last-1
Hudaldan avah last-1Hudaldan avah last-1
Hudaldan avah last-1
 
Tesviin tuhai-huuli
Tesviin tuhai-huuliTesviin tuhai-huuli
Tesviin tuhai-huuli
 
Sanhuu tesviin-zaavar
Sanhuu tesviin-zaavarSanhuu tesviin-zaavar
Sanhuu tesviin-zaavar
 
Nom900
Nom900Nom900
Nom900
 
2014 state budget toim dugnelt osf
2014 state budget toim dugnelt osf2014 state budget toim dugnelt osf
2014 state budget toim dugnelt osf
 
Aimag niislel
Aimag niislelAimag niislel
Aimag niislel
 
хүний хөгжил-сангийн-2014-оны-төсвийн-тухай
хүний хөгжил-сангийн-2014-оны-төсвийн-тухайхүний хөгжил-сангийн-2014-оны-төсвийн-тухай
хүний хөгжил-сангийн-2014-оны-төсвийн-тухай
 
нийгмийн даатгалын-тухай-хууль
нийгмийн даатгалын-тухай-хуульнийгмийн даатгалын-тухай-хууль
нийгмийн даатгалын-тухай-хууль
 

Dernier

Judging the Relevance and worth of ideas part 2.pptx
Judging the Relevance  and worth of ideas part 2.pptxJudging the Relevance  and worth of ideas part 2.pptx
Judging the Relevance and worth of ideas part 2.pptxSherlyMaeNeri
 
THEORIES OF ORGANIZATION-PUBLIC ADMINISTRATION
THEORIES OF ORGANIZATION-PUBLIC ADMINISTRATIONTHEORIES OF ORGANIZATION-PUBLIC ADMINISTRATION
THEORIES OF ORGANIZATION-PUBLIC ADMINISTRATIONHumphrey A Beña
 
Karra SKD Conference Presentation Revised.pptx
Karra SKD Conference Presentation Revised.pptxKarra SKD Conference Presentation Revised.pptx
Karra SKD Conference Presentation Revised.pptxAshokKarra1
 
Roles & Responsibilities in Pharmacovigilance
Roles & Responsibilities in PharmacovigilanceRoles & Responsibilities in Pharmacovigilance
Roles & Responsibilities in PharmacovigilanceSamikshaHamane
 
ECONOMIC CONTEXT - PAPER 1 Q3: NEWSPAPERS.pptx
ECONOMIC CONTEXT - PAPER 1 Q3: NEWSPAPERS.pptxECONOMIC CONTEXT - PAPER 1 Q3: NEWSPAPERS.pptx
ECONOMIC CONTEXT - PAPER 1 Q3: NEWSPAPERS.pptxiammrhaywood
 
Inclusivity Essentials_ Creating Accessible Websites for Nonprofits .pdf
Inclusivity Essentials_ Creating Accessible Websites for Nonprofits .pdfInclusivity Essentials_ Creating Accessible Websites for Nonprofits .pdf
Inclusivity Essentials_ Creating Accessible Websites for Nonprofits .pdfTechSoup
 
Difference Between Search & Browse Methods in Odoo 17
Difference Between Search & Browse Methods in Odoo 17Difference Between Search & Browse Methods in Odoo 17
Difference Between Search & Browse Methods in Odoo 17Celine George
 
INTRODUCTION TO CATHOLIC CHRISTOLOGY.pptx
INTRODUCTION TO CATHOLIC CHRISTOLOGY.pptxINTRODUCTION TO CATHOLIC CHRISTOLOGY.pptx
INTRODUCTION TO CATHOLIC CHRISTOLOGY.pptxHumphrey A Beña
 
Field Attribute Index Feature in Odoo 17
Field Attribute Index Feature in Odoo 17Field Attribute Index Feature in Odoo 17
Field Attribute Index Feature in Odoo 17Celine George
 
ACC 2024 Chronicles. Cardiology. Exam.pdf
ACC 2024 Chronicles. Cardiology. Exam.pdfACC 2024 Chronicles. Cardiology. Exam.pdf
ACC 2024 Chronicles. Cardiology. Exam.pdfSpandanaRallapalli
 
MULTIDISCIPLINRY NATURE OF THE ENVIRONMENTAL STUDIES.pptx
MULTIDISCIPLINRY NATURE OF THE ENVIRONMENTAL STUDIES.pptxMULTIDISCIPLINRY NATURE OF THE ENVIRONMENTAL STUDIES.pptx
MULTIDISCIPLINRY NATURE OF THE ENVIRONMENTAL STUDIES.pptxAnupkumar Sharma
 
HỌC TỐT TIẾNG ANH 11 THEO CHƯƠNG TRÌNH GLOBAL SUCCESS ĐÁP ÁN CHI TIẾT - CẢ NĂ...
HỌC TỐT TIẾNG ANH 11 THEO CHƯƠNG TRÌNH GLOBAL SUCCESS ĐÁP ÁN CHI TIẾT - CẢ NĂ...HỌC TỐT TIẾNG ANH 11 THEO CHƯƠNG TRÌNH GLOBAL SUCCESS ĐÁP ÁN CHI TIẾT - CẢ NĂ...
HỌC TỐT TIẾNG ANH 11 THEO CHƯƠNG TRÌNH GLOBAL SUCCESS ĐÁP ÁN CHI TIẾT - CẢ NĂ...Nguyen Thanh Tu Collection
 
Q4 English4 Week3 PPT Melcnmg-based.pptx
Q4 English4 Week3 PPT Melcnmg-based.pptxQ4 English4 Week3 PPT Melcnmg-based.pptx
Q4 English4 Week3 PPT Melcnmg-based.pptxnelietumpap1
 
AMERICAN LANGUAGE HUB_Level2_Student'sBook_Answerkey.pdf
AMERICAN LANGUAGE HUB_Level2_Student'sBook_Answerkey.pdfAMERICAN LANGUAGE HUB_Level2_Student'sBook_Answerkey.pdf
AMERICAN LANGUAGE HUB_Level2_Student'sBook_Answerkey.pdfphamnguyenenglishnb
 
Procuring digital preservation CAN be quick and painless with our new dynamic...
Procuring digital preservation CAN be quick and painless with our new dynamic...Procuring digital preservation CAN be quick and painless with our new dynamic...
Procuring digital preservation CAN be quick and painless with our new dynamic...Jisc
 
Keynote by Prof. Wurzer at Nordex about IP-design
Keynote by Prof. Wurzer at Nordex about IP-designKeynote by Prof. Wurzer at Nordex about IP-design
Keynote by Prof. Wurzer at Nordex about IP-designMIPLM
 
Gas measurement O2,Co2,& ph) 04/2024.pptx
Gas measurement O2,Co2,& ph) 04/2024.pptxGas measurement O2,Co2,& ph) 04/2024.pptx
Gas measurement O2,Co2,& ph) 04/2024.pptxDr.Ibrahim Hassaan
 

Dernier (20)

Judging the Relevance and worth of ideas part 2.pptx
Judging the Relevance  and worth of ideas part 2.pptxJudging the Relevance  and worth of ideas part 2.pptx
Judging the Relevance and worth of ideas part 2.pptx
 
THEORIES OF ORGANIZATION-PUBLIC ADMINISTRATION
THEORIES OF ORGANIZATION-PUBLIC ADMINISTRATIONTHEORIES OF ORGANIZATION-PUBLIC ADMINISTRATION
THEORIES OF ORGANIZATION-PUBLIC ADMINISTRATION
 
Karra SKD Conference Presentation Revised.pptx
Karra SKD Conference Presentation Revised.pptxKarra SKD Conference Presentation Revised.pptx
Karra SKD Conference Presentation Revised.pptx
 
Roles & Responsibilities in Pharmacovigilance
Roles & Responsibilities in PharmacovigilanceRoles & Responsibilities in Pharmacovigilance
Roles & Responsibilities in Pharmacovigilance
 
ECONOMIC CONTEXT - PAPER 1 Q3: NEWSPAPERS.pptx
ECONOMIC CONTEXT - PAPER 1 Q3: NEWSPAPERS.pptxECONOMIC CONTEXT - PAPER 1 Q3: NEWSPAPERS.pptx
ECONOMIC CONTEXT - PAPER 1 Q3: NEWSPAPERS.pptx
 
Inclusivity Essentials_ Creating Accessible Websites for Nonprofits .pdf
Inclusivity Essentials_ Creating Accessible Websites for Nonprofits .pdfInclusivity Essentials_ Creating Accessible Websites for Nonprofits .pdf
Inclusivity Essentials_ Creating Accessible Websites for Nonprofits .pdf
 
Difference Between Search & Browse Methods in Odoo 17
Difference Between Search & Browse Methods in Odoo 17Difference Between Search & Browse Methods in Odoo 17
Difference Between Search & Browse Methods in Odoo 17
 
INTRODUCTION TO CATHOLIC CHRISTOLOGY.pptx
INTRODUCTION TO CATHOLIC CHRISTOLOGY.pptxINTRODUCTION TO CATHOLIC CHRISTOLOGY.pptx
INTRODUCTION TO CATHOLIC CHRISTOLOGY.pptx
 
Field Attribute Index Feature in Odoo 17
Field Attribute Index Feature in Odoo 17Field Attribute Index Feature in Odoo 17
Field Attribute Index Feature in Odoo 17
 
TataKelola dan KamSiber Kecerdasan Buatan v022.pdf
TataKelola dan KamSiber Kecerdasan Buatan v022.pdfTataKelola dan KamSiber Kecerdasan Buatan v022.pdf
TataKelola dan KamSiber Kecerdasan Buatan v022.pdf
 
ACC 2024 Chronicles. Cardiology. Exam.pdf
ACC 2024 Chronicles. Cardiology. Exam.pdfACC 2024 Chronicles. Cardiology. Exam.pdf
ACC 2024 Chronicles. Cardiology. Exam.pdf
 
MULTIDISCIPLINRY NATURE OF THE ENVIRONMENTAL STUDIES.pptx
MULTIDISCIPLINRY NATURE OF THE ENVIRONMENTAL STUDIES.pptxMULTIDISCIPLINRY NATURE OF THE ENVIRONMENTAL STUDIES.pptx
MULTIDISCIPLINRY NATURE OF THE ENVIRONMENTAL STUDIES.pptx
 
HỌC TỐT TIẾNG ANH 11 THEO CHƯƠNG TRÌNH GLOBAL SUCCESS ĐÁP ÁN CHI TIẾT - CẢ NĂ...
HỌC TỐT TIẾNG ANH 11 THEO CHƯƠNG TRÌNH GLOBAL SUCCESS ĐÁP ÁN CHI TIẾT - CẢ NĂ...HỌC TỐT TIẾNG ANH 11 THEO CHƯƠNG TRÌNH GLOBAL SUCCESS ĐÁP ÁN CHI TIẾT - CẢ NĂ...
HỌC TỐT TIẾNG ANH 11 THEO CHƯƠNG TRÌNH GLOBAL SUCCESS ĐÁP ÁN CHI TIẾT - CẢ NĂ...
 
Q4 English4 Week3 PPT Melcnmg-based.pptx
Q4 English4 Week3 PPT Melcnmg-based.pptxQ4 English4 Week3 PPT Melcnmg-based.pptx
Q4 English4 Week3 PPT Melcnmg-based.pptx
 
AMERICAN LANGUAGE HUB_Level2_Student'sBook_Answerkey.pdf
AMERICAN LANGUAGE HUB_Level2_Student'sBook_Answerkey.pdfAMERICAN LANGUAGE HUB_Level2_Student'sBook_Answerkey.pdf
AMERICAN LANGUAGE HUB_Level2_Student'sBook_Answerkey.pdf
 
Raw materials used in Herbal Cosmetics.pptx
Raw materials used in Herbal Cosmetics.pptxRaw materials used in Herbal Cosmetics.pptx
Raw materials used in Herbal Cosmetics.pptx
 
Procuring digital preservation CAN be quick and painless with our new dynamic...
Procuring digital preservation CAN be quick and painless with our new dynamic...Procuring digital preservation CAN be quick and painless with our new dynamic...
Procuring digital preservation CAN be quick and painless with our new dynamic...
 
LEFT_ON_C'N_ PRELIMS_EL_DORADO_2024.pptx
LEFT_ON_C'N_ PRELIMS_EL_DORADO_2024.pptxLEFT_ON_C'N_ PRELIMS_EL_DORADO_2024.pptx
LEFT_ON_C'N_ PRELIMS_EL_DORADO_2024.pptx
 
Keynote by Prof. Wurzer at Nordex about IP-design
Keynote by Prof. Wurzer at Nordex about IP-designKeynote by Prof. Wurzer at Nordex about IP-design
Keynote by Prof. Wurzer at Nordex about IP-design
 
Gas measurement O2,Co2,& ph) 04/2024.pptx
Gas measurement O2,Co2,& ph) 04/2024.pptxGas measurement O2,Co2,& ph) 04/2024.pptx
Gas measurement O2,Co2,& ph) 04/2024.pptx
 

English syntax

  • 1. Дэд проф Ц.Сумъяа Анrли хэл егуулбэр зуй (Дурэм, да.сгал, сорихго) Гурав дахь хэвлэл Хянан тохиолдуулсан Др.Проф. Чой.Пувсснжов Улаанбаатар 2004
  • 2.
  • 3.
  • 4.
  • 5.
  • 6.
  • 7.
  • 8.
  • 9.
  • 10.
  • 11.
  • 12.
  • 13.
  • 14.
  • 15.
  • 16.
  • 17.
  • 18.
  • 19.
  • 20.
  • 21.
  • 22.
  • 23.
  • 24.
  • 25.
  • 26.
  • 27.
  • 28.
  • 29.
  • 30.
  • 31.
  • 32.
  • 33.
  • 34.
  • 35.
  • 36.
  • 37.
  • 38.
  • 39.
  • 40.
  • 41.
  • 42.
  • 43.
  • 44.
  • 45.
  • 46.
  • 47.
  • 48.
  • 49.
  • 50.
  • 51.
  • 52.
  • 53.
  • 54.
  • 55.
  • 56.
  • 57.
  • 58.
  • 59.
  • 60.
  • 61.
  • 62.
  • 63.
  • 64.
  • 65.
  • 66.
  • 67.
  • 68.
  • 69.
  • 70.
  • 71.
  • 72.
  • 73.
  • 74.
  • 75.
  • 76. Chapter Eight Exercises: Exercise 3. Through the use of computers and modems to send data over telephone lines, efficient communications is possible. 4. "Modulator and demodulator" are what modem stand for. 5. Thousands of electronic "bulletin boards" across the country is actually computers with modems that exchange data. 6. Thanks is due to volunteers called SYSOP's, or system operators, for operating those bulletin boards. 7. A network of users asks questions and checks the bulletin board. 8. Graphics are posted for others to copy for their own use. 9. A team of computer operators use modems to transmit messages at different speeds. 10. At any speed, tvelve pages take less time to send by modem than by even the speediest mail service. XOpMH rypaanyraap Aacran Exercise Twenty Three ,QapaaXb erVV1l63PVVAI.1Vir H5IrTIlaH yHwaaA, 6yxl.1Vi II anaaar sanpyynas 61.1'-1. 8ryyJ13rAXVVH, eryyJ13xVVHI.1Vi 30X1.14011A ronnoa asxaapax. A dazed crowd wander about staring at the debris. Alone on the boards of the second floor stand a toilet and sink a tub sits firmly on a concrete slab with the house in rubble around it. Two pickup trucks in front of the house rocks with their wheels in the air. The next house don't have a roof but the house after that is untouched. This sene debris and chaos are a disaster. A tomaedo has struck. No one has been killed, but hundreds of people is stunned. Fortunately, there are some relief on the way. How do the Red cross help? Within a day, a crew of caseworkers begin to interview the families and porvide assistance. Whatever the victims doesn't have any longer- clothes, as well as food and medicine- is donated and distributed. The Red Cross distribute money and vouchers rather than actual goods because that helps the local economy. Many businesses appears damaged by the disaster, and they needs customers to help them rebuild. Politics play no part here; what the Red Cross provides are help and support for all. 75
  • 77. ChapterEight Exercises: Exercise H3r A3X copanro Test One Directions: One or more of the underlined sections in the following sentences may contain errors grammar, usage, punctuation, spelling, or capitalization. Write the letter of each incorrect section, then rewrite the item correctly. If there is no error in an item, write E. Example According to Charles L. Hogue of the .=:Lo~s~_ A Angeles county Musuem of Natural Historv. "Knee-high B C to a grasshopper" is about one-half an inch. No error D E Answer B. Los Angeles County C. History, " knee-high 1.Thomas Jefferson wrote the Declaration of Independence. Which was later A B revized by Benjamin Franklin, John Adams, Congress. and Jefferson himself. C D No error E 2. The word anesthesia coined in 1846 in Boston, was formed from greek A B word parts meaning "lack of feeling," No error C D E 3. Aspirin is the most widely used drug in the world. scientists still do not ABC D know why or how it works. No error E 4, Santa Fe's official name Villa Real de la Santa de Fe San Francisco Asis A B is one of the longest place names in the world, No error C D E 5. Our sixteenth president Abraham Lincoln was voted the best U.S. A President in a 1982 survey of leading historians. political scholars and B C D authors. No error E 76
  • 78. Chapter Eight Exercises: Exercise 6. Originaly written as a bank commerical. "We've Only Just Begun" became A B C a popular hit in the 1970's. No error D E 7.Plays. written by William Shakespeare are performed throughout the world A B in such diverse languages as Japanese, Russian, , Italian and Hebrew. No C D error E 8. The twentv first century which will begin with the year 2001 is almost upon A B C us. No error D E 9. Although Poetry magazine was only a small Chicago periodical it A B contributed to the early recognition of such great poets as Frost. Pound. Eliot, C Sandburg. and Edgar Lee Masters. No error D E 10. While some people argue about whether the Loch Ness monster exists & A B all. others think there may be more than one. No error C D E 11. President Truman's full name was Harry S Truman the "S" was not an A B abreviation for a name and was correctly written without a period. No error C D E 12. Although Othello is the title role in the play of that name. The vilJian lago ABC has almost half again as many lines. No error D E 77
  • 79. Chapter Eight Exercises: Exercise Directions Each of the following sentences has a grammatical problem. On your paper write the letter of the corrected sentence. 13. The Baghdad battery is an an articraft that looks like a modern chemical battery found in the ruins of a 2000-year-old village. A. The Baghdad battery is an articraft that looks like a modern chemical battery found. In the ruins of a 2000-year-old village. B. Found in the ruins of a 2000-year-old village, the Baghdad battery is an articraft that looks like a modern chemical battery. C. The Baghdad battery is an articraft that looks like a modern chemical battery to find in the ruins of a 2000-year-old village. 14. Asking a price of $200000 in 1980, a tropical rain forest near Sea Turtle Park in Costa Rica was offered for sale. A. A tropical rain forest near Sea Turtle Park in Costa Rica was offered for sale, asking a price of $ 200 000 in 1980. B. Asking a price of $200 000, a tropical rain forest near Sea Turtle Park in Costa was offered for sale in 1980. C. Asking a price of $ 200 000, in 1980 a government agency offered a topical rain forest near Sea Turtle Park in Costa Rica for sale. 15. Appearing in 1894, Joseph Pulitzer published the first full-colur comic strip in his New York newspaper. A. Appearing in 1894 in his New Your newspaper, Joseph Pulitzer published the first full-colour comic strip. B. Joseph Pulitzer published the first full-colour comic strip in his . New York newspaper appearing in 1894. C. Appearing in 1894, the first full-colour comic strip was published by Joseph Pulitzer in his New York newspaper. 16. To put astronauts on the moon, many hours of scientific study and vast amounts of money were expended. A. Putting astronauts on the moon, many hours of scientific study and vast amounts of money were expended. B. To put astronauts on the moon, NASA expended many hours of scientific study and vast amounts of money. C. To put asronauts on the moon, vast amounts of money and many hours of scientific study had to be expended. 78
  • 80. Chapter Eight Exercises: Exercise 17. One can see the backward-growing hair that gives a Rhodesian ridgeback its name along its spine. A. The backward growing hair that gives a Rhodesian ridgeback its name can be seen along its spine. B. Along its spine, the backward-growing hair that gives a Rhodesian ridgeback its name one can see. C. The backward-growing hair that one can see gives A Rhodesian ridgeback its name along its spine. 18. Alarmed by the mild tremors, supplies were hoarded and survival kits were bought. A. Supplies, alarmed by mild tremors, were hoarded and survival kits were bought. B. Being alarmed by the mild tremors, supplies were hoarded and survival kits were bought. C. Alarmed by the mild tremors, people hoarded supplies and bought survival kits. CopMIlro Test 2 Directions One or more underlined sections in the following sentences may contain errors of grammar, usage, punctution, spelling or capitalization. Write the letter of each incorrect section; then rewrite the item correctly. If there is no error in an item, write E. Example The first money to carry the motto" In God We A B . Trust" ~ bank notes which apeared in 1864. No C D error E Answer C -was D appeared 1.Warner Brothers studios must have wanted to have set a record when it A B asked John Barrymore to bestow 191 kisses on a number of beautiful C senoritas in the film Don Juan. No error D E 79
  • 81. Chapter Eight Exercises: Exercise 2. Before his death in 1941, Robert Baden-Powell, who founded the Boy A Scouts, is also a British intelligence officer. No error BCD E 3. Many of the workers building the St. Gotthard Tunnel in Switzerland in A B C 1882 were injured. No error D E 4. The lute. with its descendents the sitar, violin, fiddle, guitar, and ukulele, A are more widely used than any other stringed instrument in the world. No B C D error E 5.Nether a large handful of potato chips D.Q[ a large bowl of pretzel have as A B C much salt as one serving of canned soup. No error D E 6. If Vincent van Gogh would have been able to sell his paintings for what A they bringtoday.he would have died a billionaire several timesover instead B C D of a pauper. No error E 7. The sphinx, a beautiful and mysterious statue, se~ not far from where A B the Great Pyramid of Giza raises into the sky. No error C D E 8. Though they could have fled before the storm began, Lee Trevino, Jerry A B Heard, and Bobby Nickols were all struck by lightning at the Western Open C D Gold Tournament in 1975. No error E 80
  • 82. Chapter Eight Exercises: Exercise 9. For camouflage a bug called Fulgora lucifera has a false head that lays A B C on the ground and resembles the head of a South American alligator. No D error E 10. Truth or Consequences were first seen in 1956, making it one of the ABC oldest game shows on television. No error D E 11. Native Americans had been living in what i§. now the United States for ABC thousands of years before Christopher Columbus "discoered" it. No error D E 12. Although virtually every man and woman in America are familiar with A B the ice-making properties of the freezer, few is aware that this mashine was C invented by a doctor to provide ice for fever-racked patients. No error D E 13. The rattlesnake and the copperhead is famous for being poisonous. but A B the cobra is the most dangerous venomous snake. No error C D E 14. In the television series,''The Many loves of Dobie Gillis," a whole cast of A Futurestars, includingWarren Beatty and Tuesday Weld, were representative B C of average American teenagers. No error D E 15. Anyone who saw the film Dr,X, has seen Humprey Bogart in a horrer ABC D movie. No error E 16.Marlon Brando was bom in Omaha Nebraska,but he don't sound like a . A B --C Midwesterner in The Godfather. No error D E 81
  • 83. Chapter Eight Exercises: Exercise 17. CaseyJones is one of thefew American folk heroswho was real and not ABC simply created by a songwriter or storyteller. No error D E 18. There § absolutely no "man-eating" plants in the world, although some ABC plants do trap insects.No error D E 19. Locusts. whichcome out of the groundonce everyseventeen years, is a A B C D serious problem in some areas of the Midwest. No error E 20.The American colonists declared there independance from Great Britain A B only after they had tried every other means to gain justice. No error C D E 21. Giant hailstones weighing more than a pound each begun to kill cattle A B as they failed on the Russian village of Kostov in July 1923: twenty-three C D people were killed trying to save the animal. No error E 22.DonQuixotemounted his horsewhile his guire, SanchaPanza, holds the ABC D animal steady.No error E 23. of one hundred crossword puzzle creators who contributed to two New York puzzle magazines in1970. one-fourth ~in prison. No error B C D E 24. Neither the members of the city council nor the chairperson~ A B C available for questions from the press. No error' D E 82
  • 84. Chapter Eight Exercises: Test copanro A 8ryyn63p yycr3x3A 3a~Ilwry~ xspsr 6oIlAor yrrnarar X3114, 3C ryxaanax X3Il4, Y~IlTH3PIo1~H 60IlOH aexuerrr yrlo1~H X3Il4, H3P rlo1wyyH eryyIl63p, Y~Il xascpan 6ylOy 6a~4 rlo1wyyH eryyIl63p rlo1wyyH, TOAOTrOIl,AaraIlAax TOAOTrOIl, MeH T3p4Il3H eryyIl63plo1~H yrlo1~H A3C napaa 33pr33p repenxces copanrsir 3H3 X3C3rT opyynas. COPIo1Ilro 6YPIo1~r xoneornox AYP3MT3~ Ys:lIlAYYIlaH a)f(Io1IlIla)f( napaa Hb eryyIl63p 6YPIo1~r caarap Hs:lrTIlaH, MOHrOIl X3IlHIo1~X33 eryyIl63p 6yr3x 3Y~ Aaryy Ha~PYYIlaH op-tyynax Hb TaHbl aHrIllo1 X3Il 60IlOH 3X X3IlHIo1~ M3AIl3rT H3H rycraa 60IlHO. CopMnroA-1 Task: Completing structure problems involving incomplete independent clauses. Directions: Choose one out of A,B,C,and D to complete the sentence. 1. In the United States, is generally the responsibility of municipal govemments. A. for water treatment B. water treatment C. where water treatment D. in which water treatment 2. Crop rotation of preserving soil fertility. A. it is one method B. one method C. a method is one D. is one method 3. the dollar as its monetary unit-in 1878. A. Canada adopted B. Adopted by Canada C. It was adopted by Canada D. The Canadian adoption of 4. almost impossible to capture the beauty of the aurora borealis in photographs. A. Being B. It is C. There is D. Is ;5. Usually political cartoons on the editorial page of a newspaper. A. appear B. whose appearance C by appearing D. when they appear 83
  • 85. Chapter Eight Exercises: Test 6. two major art museums, the Fogg and the Sadler. A. Harvard University has B. At Harvard University C Harvard University, with its D. There at Harvard University 7. American actress and direstor Margaret Webster for her production of Shakespearan plays. A. who became famous B famous as she became C becoming famous D became famous 8. gas tanks connected to welding equipment, one of full of oxigen and the other full of acetylene. A. It is two B. Of the two C. There are two D. Two 9. --,-,,..--_ is more interested in rhymtn than in melody is appearant from his compositions. A. That Philip Glass B. Philip Glass, who C. Philip Glass D. Because Philip Glass 10. Compressed air the power to drive pneumatic tools. A. by providing B. provides C. that provides D. the 'provision of 11. by cosmic rays. A. The Earth is constantly bombarded B. Bombarded constantly, the Earth C. Bombarding the Earth constantly D. The Earth's constant bombardment 12 primary colors are red, blue,and yellow. A. There are three B. The three C. Three of them D. That the three 13. =- who was elected the first woman mayor of Chicago in 1979. A. It was Jane Byrne B. Jane Byrne C. That Jane Byrne D. When Jane Byrne 14. Every computer consists of a number of systems -'-__ together. A...by working B. work C. they work D. that work 84
  • 86. Chapter Eight Exercises: Test 15. On the Moon, air because the Moon's gravitational field is too weak to retain an atmosphere. A. there is no B. where no C. no D. is no 16. The Glass Mountains of northwestern Oklahoma .with flecks of gypsum, which shine in the sunlight. A. they are covered B. covered them C. that are covered D. are covered 17. In some cases, to decide if an organism is a plant or an animal. A. difficult if B. it is difficult C. the difficulty D. is difficult 18. The first American novelist to have a major impact on world literature _ A. who was James Fenimore Cooper B. James Fenimore Cooper was C. it was James Fenimore Cooper D. was James Fenimore Cooper 19. important railroad tunnel in the United States was cut through the Hoosac Mountains in Massachusetts. A. At first B. It was the first C. The first D. As he first of 20. ·Generally, in the valleys and foothills of the Pacific Coast ranges A. the Calfornia poppy grown B. the growth of the Calfornia poppy C. the Calfornia poppy grows D. growing the Calfornia poppy 21. When bats are at rest, hang upside-down. A. they B.and C. to D.as 22. that the capital of South Carolina was moved from Charleston to Columbia. A. in 1790 was B. There was in 1790 C. in 1790 D. It was in 1790 23. Although not as important as they once were, _ _ a major form of transprotation in North America. A. there are still railroads B. railroads, which are still C. railroads are still D. railroads still being 85
  • 87. Chapter Eight Exercises: Test 24. The loop, which is the commercial heart of Chicago, _ within a rectrancular loop of electrical train tracks. A. that is enclosed B. enclosing C. is enclosed D. it is enclosed 25. _....,.--,....- amino acids that serve as the basic building blocks of all proteins. A. It was about twenty B.Forabouttwentyof C. About twenty are D. There are about twenty CopMnro A-2 Task: Answer structure problems involving incomplete adjective clause Directions: Choose one option out of A,B,C, and Dand encircle the correct word or phrase. 1. Most folk songs are ballads have simple words and tell simple stories. A. what B. although C. when D. that 2. After its introduction in 1969, the best float process the world's principal method of manufacturing flat sheets of glass. A. by which it became B. it became C. became D. which became 3. In 1850, Yale university established Sheffield Scientific School, A. engineers were educated there B. where engineers were educated C. in which were engineers educated D. where were engineers educated 4. Many of Louse Nevelson's sculptures consisted of a number of large wooden structures in complex patterns. A. which she arranged B. she arranged them C. which arranged D. arranged them 5. In addition to being a naturalist. Stewart E. White was a writer _ _ the struggle for survival on the American Frontier. A. whose novels describe B. he-describes in his novels C. his novels describe D. who, describing in his novels 86
  • 88. Chapter Eight Exercises: Test 6. Diamonds are often found in rock formations called pipes, _ _ the throats of extinct volcanoes. A. in which they resemble B. which resemble C. there is a resemblance to D. they resemble 7. William Samuel Johnson, helped write the Constitution, became the first president of Columbia College in1787. A. whom he had B. and he had C. who had D. had 8. Seals appear clumsy on the land, are able to short distances faster than most people can run. A. but they B. which they C. they D. which 9. The instrument panel of a light airplane has at least a dozen instruments A. the pilot must watch B. what the pilot must watch C. which the pilot must watch them D. such that the pilot must watch them 10. A keystone species is a species of plants or animals _ _ absence has a major effect on an ecological system. A. that its B. its C. whose D. with its 11. The size and shape of a nail depends primarily on the function in- ----:t:-ended. A. which it is B. for which it is C. which it is for D. for which is 12. In geometry, a tangent is a straight line a curve at only one point. A. it touches B. whose touching C. its touching D. that touches 13. It was the ragtime pianist Scott Joplin the Maple Leaf Rag, perhaps the best known of all ragtime tunes. A. wrote B. the writer of C. who wrote D. writing 87
  • 89. Chapter Eight Exercises: Test 14. There are 2,000 varieties of snakes, _ _are harmless to humans. A. mostly they B. most of them C. most of which D. which most 15. Smokejumpers are _ _ descend into remote areas by parachute to fight forest fires. A. firefighters B. when firefighters C. who, as firefighters D. firefighters who 16. Charlotte Gilman's best known book she urges women to become financially independent. A. is Women and Economics, in which B. Women and Economics, in which C. is Women and Economics, which D. Women and Economics, which CopMnroA·3 Task: Structure problems connected with incomplete or missing participial phrases. Directions: Choose the one option out of the A,B,C, and D to complete the sentence correctly. 1. Aerodynamics is the study of the forces on an object as it moves through the atmosphere. A. acting -B. act C. are acting D.. acted 2. for their strong fiber include flax and hemps. A. Plants are grown B. Plants grown C. Plants that grow D. To grow plants 3. ,Jose Limon's dance troupe often toured abroad. A. The U.S. State Department sponsored it B. Sponsored by the U.S.State Department C. The U.S. State Department, which sponsored it D. The sponsorship of the U.S. State Department 4. Elfreth's Alley in Philadelphia is the oldest residential street in the United States, with from 1728. A. houses are dated B. the dates of the houses C. the dating of houses D. houses dating 88
  • 90. Chapter Eight Exercises: Test 5. In,1821, the city of Indianapolis, Indiana, was laid out in a design after that of Washington,D.C. A. patterned B. was patterned C. a pattern D. that patterned 6. in front of a camera lens changes the color of the light that reaches the film. A. Placed a filter B. a filter is placed C. A filter placed D. When a filter placed 7. The Masschusetts State House, in 1798, was the most distinguished building in the United States at that time. A. completing B. which was completed C. was completed D. to be completed 8. Barbara McClintock for her discovery of the mobility of generic elements. A. known B. who knows C. knowing D. is known 9. The solitary scientist by himself has in many intances been replaced by a cooperative scientific team. A. to make important discoveries B. important discoveries were made C. has made important discoveries D. making important discoveries 10. Geometry is the branch of mathemaics the properties of lines, curves, shapes, and surfaces. A. that concerned with B. it is concerned with C. concerned with D. its concerns are 11. an average of 471 inches of rain a year, Mount Waiaeale in Hawai is the wettest spot in the world. A. It receives B. Receiving C. To receive D. Received 12. Amber is a hard, yellowish-brown from the resin of pine trees that lived millions of years ago. A. substance formed B. to form a substance C.. substance has formed D. forming a substance 89
  • 91. Chapter Eight Exercises: Test CopMnroA-4 Task: Structure problems with appositives. Directions: Choose one option that correctly complete the sentence. Mark the most appropriate one out of A,B,C, and D. 1. The Democratic party is older than the other major American political party A. which the Republican party B. the Republican party C. it is the Republican party D. the Republican party is 2. relations with friends and acquaintances, playa major role in the social development of adolecents A. What are called peer group relations are B. Peer group relations are C. Peer group relations, the D. By peer group relations, we mean 3. Joseph Henry, director of the Smithsonian Institution, was President Lincoln's adviser on scientific matters. A. the first B. to be the first C. was the first D. as the first 4. The Wasstch Range, extends from southeastem Idaho into northem Utah. A. which is a part of the Rocky Mountains, B. a part of the Rocky Mountains that C. is a part of the Rocky Mountains D. a part of the Rocky Mountains, it 5. Ruth St.Dennis turned to Asian dances to find inspiration for her choreography. A. It was the dancer B. The dancer C. That the dancer D. The dancer was 6. The organs of taste are the _ _ that are mainly located on the tongue. A. groups of cells,are taste buds B. taste buds. are groups of cells C. taste buds, these are groups of cells 90
  • 92. Chapter Eight Exercises: Test D. taste buds, groups of cells 7. In 1878, Frederick W.TayJor invented a concept called scientific management, of obtaining as much efficiency from workers and machines as possible. A. it is a method B. a method which C. a method D. called a method 8. A group of Shakers, __settled around Pleasant Hill, Kentucky,in 1805. A. members of a strict religious sect which B. whose members of a strict religious sect C. members of a strict religious sect, D. were members of a strict religious sect 9. In physics, "plasma" refers to a gas that has a nearly equal number of positively and negatively charged particles. A. the term B. by the term C. is termed D. terming 10. Norman Weiner, mathematician and logician, had an important role in the development of the computer. A. who, as a B. was a C. whom a D. a 11. Jeroma Kern's most famous work is Showboat, most endUring musical comedies. A. it is one of the finest, B. one of the finest, C. the finest one D. as the finest of the 12. a marshland that covers over 750 square miles in North Carolina and Virginia. A. In the Great Dismal Swamp B. The Great Dismal Swamp, which C. The Great Dismal Swamp, D. The Great Dismal Swamp is copanro- A-5 Task: Completing structure problems involving adverb clauses, reduced adverb clauses, and prepositional expesssions. Directions: Choose the one option out of A,B,C, and D incircling the most appropriate one 91
  • 93. Chapter Eight Exercises: Test 1. Small sailboats can easily capsize they are not handled carefully. A. but B. which C. if D. so 2. they are tropical birds, parrots can live in temperate or even cold climates. A. Despite B. Even though C. Nevertheless D. But 3. added to a liquid, antifreeze flowers the freezing temperature of that liquid. A. That B. As is C. It is D. When 4. advertising is so widespread in the United States, it has had an enormous effect on American life. A. Why B. The reason C. On account ofD. Since 5. toward shore, its shape is changed by its collision with the shallow sea bottom. A. During a wave rolls B. As a wave C. A wave rolls D. A wave's rolling 6. ____ are increasingly linked over long distances by electronic communications, but many of them still prefer face-to-face encounters. A. Although people B. Despite people C. Today people D. The fact that people 7. together in one place, they form a community. A. When people who live B. When people living C. Whenever people live D. Whenever living people 8. managed by an independent govemor and board of directors, the Bank of Canada is owned by the Canadian government. A. And yet B. In spite of it C. Although D. It is 9. pieces of rope are of different thickness, the weaver's knot can be used to join them A. Two of B. Whattwo C. Two such D. If two 10. , the seeds of the Kentucky coffee plant.are poisonous. A. Until they have been cooked B. Cooking them 92
  • 94. Chapter Eight Exercises: Test C. They have been cooked D. Cooked until 11. Natural silk is still highly prized similiar artificial fabrics. A. although is available B. despite there are available C. in spite of the availability of D. even though an availability of 12. Cattle ranches are found almost in Utah. A. whenever B. everywhere C. overall D. somewhere 13. through a prism, a beam of white light breaks into all the colors of the rainbow. A. When shines B. It shines C. It is shone D. When shone 14. most people think of freezing as a relatively modern method of food preservation, it is actually one of the oldest. A. Even B. As though C. However D. Although 15. large bodies of water never freeze solid is that the sheet of ice on the surface protects the water below it from the cold air. A. Because B. Why do C. The reason that D. For the reason 16. ---,-_ _ granted by the Patent Office, it becomes the inventor's property and he or she can keep it, sell it, or licence it to someone else. A. Once a patent is B. When a patent C. A patent, once D. A patent, whenever it 17. Owls can hunt in total darkness their remarkably keen sense of smell. A. since B. because of C. the result D. that 18. most bamboo blooms every year, there are some species that flower only two or three times a century. A. Whenever B. That C. While D. However copunro A·6 Task; Completing structure problems involving in complete noun clauses. Directions: Choose the one option out of A,B,C, and D to complete the sentences with the most appropriate words and clauses.. 1. begin their existence as ice crystals over nost of the earth seems likely. 93
  • 95. Chapter Eight Exercises: Test A. Raindrops B. If raindrops C. What if raindrops D. That raindrops 2. Scientists cannot agree on related to other orders of insects. A. that fleas are B. how fleas are C. how are fleas D. fleas that are 3. It was 1875 joined the staff of the astronomical obserbatory at Harvard University. A. that Anna Winlock B. Anna Winlock, who C. as Anna Winlock D. Anna Winlock then 4. is a narrow stop od woods along a stream in an open .grassland. A, Ecologists use the term "gallery forest" B. What do ecologists call a "gallery forest" C. "Gallary forest" is the term ecologists use D. What ecologists call a "gallary forest" 5. developed so rapidly in Alahama primarily because of its rich natuarl resources. A. That heavy industry B. Heavy industry C. Heavy industry that was D. When heavy industry 6. _ _...,.,- so incredible is that these insects successfully migrate to places that they have never even seen. A. That makes the monarch butterflies' migration B. The migration of the monarch butterflies is C. What makes the monarch butterflies' migration D. The migration of the monarch butterflies, which is 7. Art critics do not all agree on what a painting great. A. qualities make B. are the qualities for making C. qualities to make D. do the qualities that make 8. In order to grow vegetables properly, gardeners must know A. what the requirements TOr each vegetable are B. that the requirements for each vegetable C. what are each vegetable's requirements D. that is required by each vegetable 9. When is not known. A. was the wheel invented B. the invention of the wheel 94
  • 96. Chapter Eight Exercises: Test C. inventing the wheel D. the wheel was invented 10. For many years people have wondered exists elsewhere in the universe. A. that life B. life which C, whether life D. life as it 11. of all modern domestic poultry is the red jungle fowl is widely believed. A. The ancestor B. The ancestor is C. How the ancestor D. That the ancestor 12. the right side of a person's brain is dominant, that person is left- handed. A. That B. If C. Which D. For copanro A·7 Task; Completing structure problems involving word order. Directions: Choose the one option-A,B,C, or D-that correctly completes the sentence, then mark the appropriate blank. 1. Hills known as land islands, or salt domes, are Louisiana's marshlands. A. extremely interesting features of B. of extremely interesting features C. interesting extremely features of D. extremely interesting of features 2. of chamber music is the string quartet. A. The famous most from B. The most famous form C. The form most famous D. Most the form famous 3. Not until the seventeenth century to measure the speed of light. A. did anyone even attempt B. anyone did even attempt C. did anyone attempt even D. did even attempt anyone 4. Alfalfa is for livestock. A. a primarily grown crop B. grown primarily a crop C. a crop grown primarily D. a grown crop primarily 95
  • 97. Chapter Eight Exercises: Test 5. The Franklin stove, which became common in the 1790's, burned wood _ _ an open fireeplace. A. efficiently more than much B. much more efficiently than C. much more than efficiently D. more efficiently much than 6. Reinforced concrete is concrete that is strengthened by metal bars.--:-__ A. in it that are embedded B. embedded that are in it C. are that it embedded in D. that are embedded in it 7. The type of clothing people wear tells others a lot about _ A. who they are B. are they who C. they are who D. who are they 8 Most southern states had set up primary school systems by the late eighteenth century, but only in New England and open to all students. A. primary schools were free B. were primaary schools free C. free were primary schools D. were free primary schools 9. Fungi, , do not produce chlorophyll. A. as such mushrooms B. mushrooms as such C. such as mushrooms D. mushrooms such as 10. Seldom --:---::-_ more than 20 minutes a night. A. sleep giraffes B. do giraffes sleep C. giraffes do sleep D. giraffes sleep 11. of the early years of space exploration was the discovery of the Van Allen radiation belt in 1958. A. Perhaps the greatest triumph B. The triumph perhaps greatest C. The greatest perhaps triumph D.The triumph greatest perhaps 12. Today major new products without conducting elaborate market research. A. corporations hardly introduce ever B. hardly ever corporations introduce C. hardly ever introduce corporations D. corporations hardly ever introduce 13. Across the Chesapeake Bay from the rest of the state _ whose farms produce beans, tomatoes, and garden vegetables. A. there lies Maryland's Eastern Shore 96
  • 98. Chapter Eight Exercises: Test B. lies Maryland's Eastern Shore C. Maryland's Eastern Shore lies there D. Maryland's Eastern Shore lies 14. Acidophilus bacteria are ~_ _ in an acid medium. A. those that grow best B. those grow best that C, that those grow best D. grow best those that 15, of great apes; the gibbon is the smallest. A. Four of the types B, The four of types C, Four types of the D, Of the four types 16, It is difficult through swamps because of tangled roots and shallow waterways. A. to navigate even for small boats B. for even small boats to navigate C. Even small boats for to navigate D. even to navigate for small boats 17. A lodestone is -,,- _ A. an occurring naturally magnet B. a magnet naturally occurring C. naturally a magnet occurring D. a naturally occurring magnet 18, So complicated that consumers who use a product are seldom aware of where all its components come from. A. today trade is international B. today international trade is C. is international trade today D, international trade is today 19. The snow bunting is winter birds in Canada. A. one most of the common B. the most common one of C. one of the most common D, the one of most common 20, Nashville has the capital of country music, A. as long been known B, been known long as C. long been known as D. long as been known copanro A- 8 Task: Completing structure problems involving infinitive and gerund phrases, Directions: Choose the one option out of the A,B,C, and D to complete the sentences with the most appropriate ones, 97
  • 99. Chapter Eight Exercises: Test 1. for a career in dance generally begins at an early age. A. People train 2. That people train C. If training 4. Training 2. A baby's first teeth are generally the lower incisors. A. appearance B.appear C. to appear D. in appearing 3. A climbing helmet protection for a rocket-climbers head from falling rocks and other hazards. A. to provide B. provides C. providing D. that provides 4. Power tools require careful handling injuries. A. by avoiding B. they avoid C. to avoid D. that avoid 5. An electromagnet is created electrical current through a coil of wire A. by passing B. passes by C. to be passed D. passed 6. at home requires only three types of chemicals, several pieces of simple equipment, and running water. A. For the development of film. B. To develop film C. When film is developed D. In developing film 7. The purpose of cost accounting is involved in producing and selling a good or service A. as a determination of its costs B. the costs determined C. that determines the costs D. to determine the costs 8. was one of the most difficult tasks pioneers faced on their joumeys west. A. Crossing rivers B. While crossing rivers C. Rivers being crossed D. By crossing rivers 9. Energy can be defined as the ability :--:----:__ A. do working B. to do work C. doing work D. work to be done 10. The process of _-_ _ by hand has changed little since the fifteenth century. A. to bind books B.binding books 98
  • 100. Chapter Eight Exercises: Test C. books are bound D. bound books 11. A crescent wrench has adjustable jaws for a nut, bolt, or pipe. A. to grip B. they grip C. gripping D. gripped 12. Compressed air is air brakes, pneumatic tools, and other machinery. A. used to powering B. to use powering C. used to power D. in use by powering 13. Some people believe that the crystals of certain minerals _ curative powers. A. have B. having C. that have D. to have 14. The narrow blades of speed skates allow _ _ speeds of up to 30 miles per hour. A. for skaters maintaining B. skaters to maintain C. skaters maintain D. maintenance by skaters 15. The first library _ _ in the Nebraska Territory was built in Fort Atkinson in 1870. A. to be established B. was established C. could establish D. to establish copanro A-9 Task: Completing structure problems involving paralleism. Directions: Choose the one option out of the A,B,C, and D to complete the sentence with the most appropriate one. 1. Insects provide many beneficial services, such as _ breaking down deadwood, and pollinating plants. A. they condition soils B. to condition soil C. conditioning the soil D. soil conditioned 2. Frozen orange juice must be packed, and stored when the fruit is ripe. A. be frozen B. must be frozen C. frozen D. it must be frozen 3. The Sioux language is spoken not only Sioux but also by the Crow and Osage tribes. A. by the B. the C. do the D. and the 99
  • 101. Chapter Eight Exercises: Test 4. In 1900 electrically powered cars were more popular than gaseline· powered cars because they were quiet, opreated smoothly, and ~--,-_ A. handled easily B. ease of handling C. handling easily D. easy to handle 5. Roger Williams was a clergyman, the colony of Rhode Island, and an outspoken advocate of religious and political freedom. A. founded B. the founder of C. was the founder of D. he founded 6. Paint can be applied to a surface with rollers, _ _ , or spray guns. A. brushes B. brushes can be used C. with brushes D. by brush 7, The use of labor-saving devices in homes, , and in factories added to the amount of leisure time people had. A. at office B. used in offices C. offices D. in offices 8. A dulcimer can be played by either striking its strings with a hammer or.,..-,.--,.,.._' A. to pluck them with the fingers B. fingers are used to pluck them C. they are plucked with the fingers D. plucking them with the fingers 9. Throughout history, trade routes have increased contact between people, , and greatly affected the growth of civilzation. A. have resulted in an exchange of ideas B. an exchange of ideas has resulted C. resulted in an exchange of ideas D. resulting in an exchange of ideas 10. Walt Disney made many technical advances in the use of sound, color, and in animated films. A. photographing B. using photography C. photography D. use of photographs 11. Artist Paul Kane traveled throughout Northwest Canada on foot, by canoe, and to sketch Native Canadians going about- their ordinary lives. A. on horseback B. riding a horse 100
  • 102. Chapter Eight Exercises: Test B. was on horseback D. by a horse 12. Barbara Jordan was the first woman in the South to win an election to the House of Representatives, as Congresswoman from Texas from 1973 to 1979. A to serve B. served C. serving D. has served 13. Photographers' choice of a camera depends on what kind of pictures they want to take, how much control they want over exposure, and they want to spend. A the amount of money B. what money C. how much money D. so much money that 14. Atlanta is the commercial, financial, and of Georgia. A center of administration B. administrative center C. center for administering D. administrating center 15. Even after the Revolutionary War, American importers obtained merchandise from Britain because British merchants understood American tastes, offered attractive prices, and -:-__ A easy credit was provided B. because of easy credit C. easy credit D. provided easy credit CopMnro A ·10 Task: Completing structure problems involving prepositional phrases. Directions: Choose the one option out of of A,B,C, and D, correctly completing the sentence. 1. seed of a flowering plant is covered by a dense . protective coat. AGn each B. Each C. Each of D. That each 2. Dynamite is ordinarily detoned called a blasting cap. A a device is used B. that a device C. with a device D. the use of a device 101
  • 103. Chapter Eight Exercises: Test 3. 1900 there were some 300 bicycle factories in the United States and they produced over a million bicycles. A. In B. Because in C. It was in D. That in 4. A thick layer of fat called blubber keeps whales warm even _ coldest water. A. although the B. in the C. the D. of the 5. the United States, the general movement of air masses is from west to east. A. Across B. To cross C. They cross D.lt's across 6. The bark of a tree thickness _ A. with age B. it gets older C. as older D. by age 7. A substance that is harmless to a person who has no allergies can cause mild to serious reactions in a person allergies. A. has B. which having C. can have D. with 8. In 1868 a number of national unions formed the American Federation of Labor ..,.- A. Samuel Gompers was its leader B. under the leadership of Samuel Gompers C. which, under Samuel Gompers' leadership D. Samuel Gompers led it 9. Harmonicas. autoharps, and kazoos folk intruments. A. are examples B. for example C, are examples of D. as examples of 10. charming shops and restaurants, Old Town is the most picturesque section of Albuquerque. A. With its B. Its C. Because its D. For its 11. such as banking and travel, in which computers are not a convenience but a necessity. A. Where some industries, B. In some industries C. Some industries D.There are some industries, 12. One of the oldest large suspension bridges still today is the George Washington Bridge between New York City and Fort Lee, New Jersey A. uses B. is used C. the use of D in use cepanro A -11 Task: Completing structure problems involving misplaced modifiers. 102
  • 104. Chapter Eight Exercises: Test Directions: Choose the one option out of A,B,C, and D and complete the sentence correctly. 1. Fearing economic hardship, _-:-----:---:-:~:--':-:-:'"-;-- A. many New Englanders emigrated to the Midwest in the 1820s B. emigration from New England to the Midwest took place in the 1820s C. it was in the 1820s that many New Englanders emigrated to the Midwest D. an emigration took place in the 1820s from New England to the Midwest. 2. Rich and distinctive in flavor:::-: _ A. there is in the United States a very important nut crop, the pecan B. the most important nut crop in the Unites States, the pecan C. farmers in the United States raise pecans, a very important nut crop D. pecans are the most important nut crop in the United States 3. Orbiting from 2.7 to 5.6 billion miles from the sun ",=",_....,...-....,. A. the astronomer Clyde Tombaugh discovered Pluto in 1930 B. Pluto was discovered by the astronomer Clyde Tombaugh in 1930 C. it was in1930 that the astronomer Clyde Tombaugh discovered Pluto D. the discovery of Pluto was made by Clyde Tombaugh in 1930 4. A popular instrument --:-_.,....-_ A. only a limited role has been available to the accordion in classical music B. there is only a limited role for the accordion in popular music C. classical music provides only a limited role for the accordion D. the accordion has played only a limited role in classical music 5. Unlike most birds, A. the heads an-d-:--nec--:-k-s of vultures lack feathers B. feathers are not found on the heads and necks of vultures C. vultures do not have feathers on their heads and necks D. there are no feathers on vultures' heads and necks 6. Widely reproduced in magazines and books, ..,...-_----:--:-:_' A. Ansel Adams depicted the Western wilderness in his photographs B. the Western wilderness was depicted in the photographs of Ansel Adams C. Ansel Adams' photographs depicted the Western wilderness D. it was through his photographs that Ansel Adams depicted the WesterQ. wilderness 103
  • 105. Chapter Eight Exercises: Test 7. Smaller and flatter than an orange, .-:-:-_---._ A. a tangerine is easy to peel and its sections separate readily B. the peel of a tangerine is easily removed and its sections are raedily separated C. it's easy to peel a tangerine and to separate its sections D. to peel a tangerine is easy, and its sections can be readily separated 8. Like the federal government,~--:----:~ A. taxation provides most of the funds for state and local governments as well B. state and local governments obtain most of thier funds through taxation C. through taxation is how state and local governments obtain most of their funds D. funds are provided from taxation for state and local governments 9. Originally settled by Polyesians around 700 AD, _ A. Hawaii received its first European visitor in 1778, when Captain James Cook landed there B. Hawaii's first European visitor, Captain James Cook, landed there in 1778 C. in 1778 the first European, Capatain James Cook, visited Hawaii D. the first European to visit Hawaii was Captain James Cook, landing there in 1778 10. Unlike most modernist poets, based on ordinary speech. A. Robert Frost's poems were B. the works of Robert Frost were C. Robert Frost wrote poems that were D. the poetry written by Robert Frost was 11. Named for its founder, in Ithaca, New York. A. in 1.865 Ezra Cornell established Cornell University B. Cornell University was established in 1865 by Ezra Cornell C. it was 1865 that Cornell University was established by Ezra Cornell D. Ezra Cornell established Cornell University in 1865 12. While living in New Orleans, the Creole people of Louisiana. A. a book of folklore, Bayou Folk, was written by Kate Chapin about B. Bayou Folk, a book of folklore, was written by Kate Chapin about 104
  • 106. Chapter Eight Exercises: Test C. the subject of Kate Chapin's book Bayou Folk was the folklore of D. Kate Chapin wrote Bayou Folk, a book about the folklore of copanro 6 ,QapaaXb copanrur TyxaH TyxaHH 6yr31., AaanraBapblH Aaryy axannax Hb TaHbl X3n 3yHH. TyxaHn6an. eryyn63p 3YHH TyxaH M3Afl3r oanrouur 6aTaraxblH 33p3r1.33r33p eryyn63p 6YPMHH WMH3 yr X3nn3rMHr caHH rons 6101'·lr33c caarap HRrTnaH Y33>K, copanrur ryH1.33X X3P3rT3H. copanrur a>KMnnaCHbI napaa eryyn63p TyC 6ypMHr 6yr31.. repen, eryyn63p ,Q3X yrMHH A3C Aapaa, eryyn63pMHH rMwYV,QMHH 6aHpnanblH XYBb,Q AaXMH ryuraaa Y33>K 3X x3nHMHx33 X3B X3M>K33HMH Aaryy opayynax Hb TaHbl aHrnM x3nHMH M3Afl3r op-yynax yp Ya,QBapT TyC 60nHo. CopMnro 6·1 Choose the correct answer for the blank. When you choose the correct answer pay attention to the use of the verbals. 1. One of the professor's great attributes is ' A. when he gives lectures B. how in the manner that he lectures C. the wat to give lectures B. his ability to lecture 2. To get an education, _-:- _ A. one must work hard B. working hard is one of the most requirements C. requirement is needed to work hard D. working hard is needed 3. John said that he had run in order _ A. that he catch the bus B. that he can catch the bus C. to catch the bus D. to the bus he could catch 4. "lVhy was the official meeting called?" "Yes new officers." 105
  • 107. Chapter Eight Exercises: Test A Select B. Selecting C. To select D. For selecting 5. "Where did he go?" " He went to another store.--,- _ A to buy slacks B. for buying slacks C. buy slacks D. buying slacks 6. "I'll help you whenever you need me." "Good. I'd like __ me tomorrow." A you helping B. that you will help C. you to help D. that you help 7. "The photographer thought he was supposed to arrive after the quests." " But I meant early." A for his coming B. him coming C. for him to come D. he would come 8. " My baby has an infection.."" Did the doctor find it difficult _ _?" A. in treating B. treating C. for treating .. D. to treat 9. "What's made Ruth so upset?" "_ _ three tickets to the folk music concert." A. Lost B. Losing C. Because of losing D. Since she lost 10. "What will Andrew be doing in the fall?" " mathematics at a private boys' school." A. To teach B. Teaching C. will be teaching D. will teach 11. The company manager may enable the men who tend the machines_ _ a large panorama of possibilties. A to see B. see C. Seeing D.seen 12. When they met, Leonardo and his enemy were fighting > A. killed each other B. killing each other C. to be killed each other D. to kill each other 13. The president of a company should know - A. to be firm B. the way of firmness C. to have firmness D. how to be firm 14. "Why were you so late for work today?" " to the office was very slow this moming because of the traffic." A Driving B. To drive C. I drove D. That II drove 15. time and labor, cartoonist generally draw the hands of their characters with only three fingers and a thumb. A Saved B. Saves C. To save D. The saving 106
  • 108. Chapter Eight Exercises: Test 16. Lincoln, Nebraska, is an important manufacturing, insurance, and A. shipping of grain B. to ship grain C. grain was shipped D. grain-shipping 17. There is no what may happen. A. know B. knowing C. to know D. known 18. "How did you spend the night?" "We had a very hard time _ _.,-some of the problem." A. discussing B. to discuss C. of discussing D. on discussing 19. With her do this, she will have no difficulty persuading them to accept her plan. A. I help B. my helping C. me helping D. mine helping 20. "What are you going to do tomorrow?" "We are going _ _ tomorrow." A. to climb B. climbing C. climbed D. climb 21. She must be looking forward as much to his return as he himself is to her. A. see B. have been C. seeing D. having seen 22. She __ help thinking that she had seen him somewhere before. A. can not B. could not C. must not D. might not 23. He has lots of books, that he is still young. A. considering B. considered C. being considered D. our considering 24. "What would you like for you birthday?" "I'd like the __ works of Maugham." A. collected B. collection B. collecting D. collect 25. The peart is the only gem by a living creature. A. creating B. to create C. is created D. created 26. to steel, chrominum increases the metal's hardness. A. Added B. In addition B. Adding D. Adds 27. Commercial banks make most of their income from interest _ _.,- on loans and investments in stocks and bonds. A. earn B. earned C. to earn D. was earned 107
  • 109. Chapter Eight Exercises: Test 28. "What did they think about the plan?" "Everyone was so happy and about it." A. excite B. exciting C. excited D: excites 29. "How do you like coffee?" "llike coffee _ A. that is sweetened B. sweetened C. being sweetened D. to be sweetened 30. "How did you learn to drive?" " _ _ strict obedience to my tutor." A. Giving B. Give C. to give D. By giving 31. After taking his examination, _ A. the book was read by him B. the book made him happy to read it. C. he wanted to read the book D. the reading of the book gave him some pleasure 32. Anesthetics are used insensitivity to pain during surgical operations. A. the cause B. to cause ' C. cause of D. causing J 33. Ganga went to the nicest store in the city _ _ presents for her children. A. for to get B. for getting C. to get D. to getting 34. "Are you ready now?" "Yes, Let's hurry _ _ a seat near the front." A. so to get B. for getting C. get D. toget 35. At last we found the exhausted animal lying there, _ A. sick B. to be sick C. in sick D. to be sickening Written Expression Directions: The four underlined parts of the sentence are marked A,B,C and D. Identify the one underlined word or phrase that must be changed in order for the sentence to be correct. 36. Twenty to thirty year after a mature forest is cleaned away, a nearly A B C impenetrate thicket of trees and shrubs develops. D 37. The first natural in world, Bogdo Uul Reserve Park, was established in 108
  • 110. Chapter Eight Exercises: Test A B C D 1772. 38. Becauseit doeshavea blood~, the comeatakestheiroxygen direcUy A B C D from the air. 39. Magnificent mountains and coastal scenery 12 British Columbia's chief ABC D tourist attrations. 40. Scientists at universities are often more involved in theoritical research ABC than in practically research. D 41. Nylon, a synthetic done from a combination of water, air and a by-product of coal, was first introduced in 1938. 42.0mithology, the study of birds, is one of the major scientific fields in which A B amateur ~ a role in accumulating, researching, and publish data. C D 43. Animation is a technigue for creativity the illusion of life in inanimate things. ABC D 44. On December 7, 1787, Delaware became a first state to ratify the A B C D Constitution. 45. Nutritionists believe what diet affects how one feels physically and ABC D emotionally. 46. Stars in our universe vary in temperature, color, bright, size and mass. A B C D 47. Ice is less denser than the liquid from which 11 is formed. ABC D 48. The 1983 Nobel Prize in Medicine was awarded to Barbara McClintock for A her experiments with maize and her discoveries regardless the nature of B C D DNA. 49. ill 1866 to 1883, the bison population in North American was reduced A B from an estimated 13 million to a few hundred. C D 50. Most of the damage property attributed ill the San Francisco earthquake A B Qf 1906 resulted from the fire that followed. C D 109
  • 111. Chapter Eight Exercises: Test copanro 6-2 The test is related with a complex sentence or clauses: a main clause and a subordinate clause. So, before doing the test you may review the clauses. Choose the correct one out of A,B,C, and 0 to complete the sentences correctly. 1. some mammals came to live in the sea is not known. --p:.-Which only brown thrashers B. Since C. Although D. How 2. all behavior is learned behavior is a basic assumption of social scientists. A. Nearly B. That nearly C. It is nearly D. When nearly 3. George _ _ he could improve his test scores, but he did not have enough time to study. A. knew to B. knew how C. knew how that D. knew how to 4. The people at the party were worried about Janet Because no one was aware she had gone. A. where that B. of where C. of the place where D. the place 5. progress helps to relieve scarcities is a fact accepted by economists. A. Technological B. That technological C. Although technological D. Ther is technological 6. "Do you remember where _ _ my watch?" A. had I put B. had put I C. I had put D. put I 7. I wonder how many years ago _ _. A. did your father retire B. your father retired C. has your father retired D. your father has retired 8. "Suie has't written us for such a long time." A. that she happen B. happened C. to happen D. having happened 9. There are very few areas in the world be grown successfully. A. where apricots can B. apricots can C. apricots that can D. where can apricots 10. Caves and hollow trees are not the only places ......,.,._-,-_ A. where do bats live B. bats live where C. where bats live D. live where bats 110
  • 112. Chapter Eight Exercises: Test 11. "May I have the loan?" " _ _ you offer good security." A. But B. Unless C. Provided D. But for 12. ---:----=::--" heat is produced. A. The mixing together of certain chemicals B. Whenever certian chemicals are mixed together C. Certain chemicals mixed together D. That certain chemicals are mixed together 13. However much , it will be worth it. A. does the watch cost B. costs the watch C. the watch will cost D. the watch costs 14. Gorillas are quiet animals, they are able'to make about twenty different sounds. A. how B. in spite of C. because of D. even though 15. that the formation of the sun, the planets, and other stars began with condensation of an interstellar gas cloud. A. Believing B. To believe C. The belief D. It is believed 16. sandstone is broken apart, it is usually the cementing material that fractures. A. a B. in a C. It is a D. When a 17. _ _ is indispensible to plant and animal life. A. Nitrogen B. It is nitrogen C. That nitrogen D. Although nitrogen 18. As soon as the gate opened, -:--_ A. we found that the two men emerged B. the two men had emerged C. here the two men emerged D. the two men emerged 19. The first doll say "mama" was invented in 1830. A. that it could B. could it C. it could D. that could 20. relatively costly, the diesel engine is highly efficient and needs servicing infrequently. A. Even B. It is C. Even though D. There is 21. We might still catch the train if we -:---:-----:-_ A. make hurry B. haste C. make haste D. hastily 22. Almost all economists agree by trading with one another. A. nations that are gained B. nations they gain C. gaining nations D. that nations gain 111
  • 113. Chapter Eight Exercises: Test 23. dog was the first animals to be domesticated is generally agreed upon by authorities in the field. A. Until the B. It was the C. The D. That the 24. kinds of dinosaurs were dying out all through the Age of Reptiles is true. A. Some B. When some C. Some were D. That some 25. On the side of the hill, there is a which was once the park. A. deep hole in the ground B. hole deep in ground C. deep hole in the ground D. deep in the ground hole 26. within the algae began very early is evident. A. Evolution that was B. That evolution C. There was evolution D. It is estimated that 27. All of the plants now raised on farms have been developed from plants wild. A. once they grow B. they grow once C. that once grow D. once grow 28. Seeds usually germinate the temperature is favorable. A. If B. whereas C. as a result D. in consequences 29. The knee is the joint the thigh bone meets the large bone of the lower leg. A. when B. where C. why D. which 30. The costs of distribution and sales make up a large part of the prices that _-:-:--:-- A. are paid for all products B. all products are paid for C. for which all products are paid D. for all products paid 31. Neon is said to be inert _ _ does not react easily with other substances. A. because of it B. because it C. it is because . D. is because it 32. ___" glasses can correct most sight defects in healthy eyes. A. When well fitted B. Well fitted when C. Well fitted if D. If well fitted when 33. he has created striking stage sittings for the Martha Graham dance company, artist Isamu Noguchi is more famous for his sculpture. A. But not B. Nevertheless C. In spite of D. Although 34. A logarithm is ~__ in algebra as an exponent. A. known what B. known what it is C. what is known D., what it is known 112
  • 114. Chapter Eight Exercises: Test 35. Although many colonial scholars consider Joanthan Edwards important writer, any more. A. though few people read his works B. but few people read his works C. and his works are not widely read D. his works are not widely read 36. The averagetemperature on Mars,the foothplanet from the sun,is about A B C eighty degrees than colder on Earth. D 37. One of the longest wars in historywere the Hundred Years War, fought A B C between England and France in the fourteenth and fifteenth centuries. D 38.Any material that l§. attractive by a magnet is by definition magnetic" ABC D 39.What are common known as "Iead"pencils are not lead. but rather a A B C mixture of graphite, clay and wax. D 40. In the last two decades, Bombay and Madras are developed into centers of A B C the Indian film Industrv. D 41. The abilities to work hard, follow directions, and thinking indepedently are A B C some of the criteria for success in the work place. D 42. The average cat sleeps sixteen hours for a day, in short intervals called" cat naps" D 43. The elephant relies more on its sense of smell than for any other sense. ABC D 44. Some insects hear ultrasonic sounds more than two octaves than higher 113
  • 115. Chapter Eight Exercises: Test A B C D humans can. 45. To stay warm in cold weather, cold-blooded animals must expose itself to g A B source of warmth such as direct sunlight. C D 46. In the wild, tea plants become trees of approximately thirty feet high. A B C D 47. Accounting is described as art of classifying, recording, and reporting A B significant financial events. C D 48. The development of the watch depended upon the invent of the mainspring. A B C D 49. Physical fitness activities can lead to an alarming variety of injuries if A B participants push themselves greatly hard. C D 50. The structure but behavior of many protozoans are amazingly complex for A B C single-celled animals. D copanro 6-3 This test is related with some Confusing Verbs. When you do the test pay attention to the Confusing Verbs. Choose the correct one out of A,B,C, and B. 1. Before discussing the steps in detail, -,--,-__ A. a general principle should be laid down B. I should like to lie down a general principle C. I'd like to lay down a general principle D. a general principle lies 2. The typewriter _---,,,--..,........, A. Was laying on the table, where it had laid all week 114
  • 116. Chapter Eight Exercises: Test B. was lying on the table, where it had laid all week C. was lying on the table, where it had been laid all week D. was lying on tha table where it had been laid all week 3. "What dis Joe do?" "He asleep all morning. A. lain B. laid C. lay D. lying 4. Up till then, these problems had been for centuries. A. laying dormant B. laying dormantly C. lying dormant D. lain dormantly 5. During the period of inflation, the value of money drops as A. prices rise B. prices arise C. the price is rising D. prices are raised 6. "Prices keep going up". " Yes, but the interest on my savings account is certainly not " A. risen B. raise C. on the rise D. on the raise 7. "When did the lecture begin?" "When all the students _ the professor began his lecture. A. seated B. sit B. were seated D. seat 8. My uncle says that:_ _-:c--_ A. my aunt sat her suitcase in corner B. he rises at five o'clock every morning C. the river had raised two feet during the night D. from where he was lying, he could see a small black box setting on the table. 9. It is said that _ A. for six months the factory machinery has laid idle B. I finished laying the bricks, but they did not lay evenly C. my mother immediately set the kettle on the stove D. we must have setr three hours waiting for him 10. Most of the begining students cannot understand what Professor Majid A. talks B. says about C. says D. discusses about 11. "My watch five o'clock." "It's very late. I must go". A. says B. tells C. speaks D. talks 12. "What do you need?" "Could you me ten dollar?' A. borrow B. borrowing C. lend D. lending 13. "What do you need?" "My pen is out of in!c, may I ? A. use yourrs pen B. lend yours 115
  • 117. Chapter Eight Exercises: Test C. borrow yours D. will borrow 14. When you go to the library tomorrow, _ _-.,.-_ A. please go to check the reference room if Mary were there B. please take this book to Mary C. please bring the note to the librarian D. please bring this book to the librarian in the reference room 15. "Oh, glad to see you, John." "What you to this big city? Be here long? A. carries B. brings C. takes D. makes 16. "Why do you like that place?" "I _ _in that city." A. bome B. had bomed C. bear D. was born - 17. "He has been working very hard recently." "Because the burden of a big family has been on his shoulder." A. bear B. borne C. born D. bore 18. "What does the judqe want me to do?" "He wants you to, _ A. sayan oath B. tell on oath C. do an oath D. take an oath 19. The visiting delegates were urged to ,_:-:---:-:----:- A. talk at library B. state their open minds C. make individual expressions D. speak freely 20. He me all about his trip. A. said B. talked C. talks D. tells 21. 'What haopenedtothe leader of the mob?"" He was_at down." A. hang B. hung C. hanged D. hanging 22. The teacher has the story many times. A. told B. stated C. said D. talked 23. While searching for the wreckage of s unidentified aircraft, the Coast ABC Guard encountered severe squalls at sea. o 24. Although a number of police officers was guarding the priceles treasures in A .13 116
  • 118. Chapter Eight Exercises: Test the museum, the director worried that someone would try to steal them. C D 25. Since it was so difficult for American Indians to negotiate a peace treaty or A B declare war in their native language, they used a universal understood C D form of sign language. 26. While verbalisation is the most common form of language in existance, A B humans make use of many others systems and techniques to express C D their thoughts and feelings. 27. The need for a well-rounded education was an idea expoused by the A B C Greeks in time of Socrates. D 28. Writers and media personnel sell theirsetves best by the impression given ABC in their verbal expression. D 29. In the spirit of the naturalist writers, that author's work portrays man's A B C struggle for surviving. D 30. Stephen Crane's stOry is .§ clinical portral of man as an animal trapped by A B C the fear and hunger. D 31. Their silly, whiny convesation on a child level was meant to create tension A B and heighten Nancy's fears and anxiety. C 0 117
  • 119. Chapter Eight Exercises: Test 32. For a long time, this officials have been known throughout the country as A B C politcal bosses and law enforcers. D 33. The development of the watch depended upon the invent of the A B C D mainspring. 34. The ordeal of the Cherokee Indians, who were forcible moved from their A B homeland in the 1830's, is remembered as the "Trail of Tears." C D 35. Physical fitness activities can lead to an alarming variety of injuries if A B participants push themselves greatly hard. C D 36. The structure but behavior of many protozoans are amazingly complex A B C for single-celled animals. D 37. Alaska's rough climate and terrian divide the state into isolated regions, A B making highway maintainance difficulty. C D 38. For hundreds of years, sailors relied on echoes to warn them of another A B C ships, icebergs, or cliffs in fQggy weather. D 39. Although he is employed in the scientific and technical fields, the metric A B system is not generally utilized in the United States. C D 40. Nora hardly never misses an opportunity to.l2ill.v in the tennis tournaments. A B C D 118
  • 120. Chapter Eight Exercises: Test 41. Air pollution, together with littering, are causing many problems in oor ABC D large, industrial cities today. 42. Because of the severe snow storm and the road blocks, the air force A B dropped food and medical supplies close the city. e D 43. Hummingbirds are the only birds capable to fly backward as well as ABC forward, up, and down. D 44. The news of the president's treaty negotiations with the foreign government A were recieved with mixed emotions by the citizens of both governments. B C D 45. Angie's bilingual ablitity and previous experience were the qualities that A which helped her get the job over all the other candidates. B e D 46. Joel giving up smoking has caused him to gain weight and become irritable ABC D with his acquaintances. 47. They asked me what did happen last night, but I was unable to tell them. A B C D 48. The test administrator ordered we not to open our books until he told us to A B C D do so. 49. Our new neighbors had been living in Arizona since ten years before A B C D moving to their present house. 119
  • 121. Chapter Eight Exercises: Test 50. I would of attended the meeting of the planning committee last week, but! A B had to deliver a speech at a convention. C D ccpanro 6-4 Structure and Written Expression Choose the one word or phrase that best completes the sentence. 1. The cyclist he crossed the main street. A. looked with caution after B. had looked acutiously before C. was looked cautous when D. looks cautious when 2. Here notebook and report that I promised you last week. A. is the B.are the C. was the D. has been a 3. Neither Jane nor her brother a consent form for tomorrow's field trip. A. need B. needs C. is needing D. has need 4. Cuba is sugar-growing areas in the world. A. one of the larger B. one of largest C. one of the largest D. largest 5. The skiers would rather through the mountains than go by bus. A. to travel on train B. traveled by train C. travel by train D. traveling by the train 6. That magnificent temple was constructed by the Chinese. A. eight-centuries-old B. eight-century's-old C. old-eight-centuries D. eight-century-old 7. There were two small rooms in the beach house, served as a kitchen. A. the smaller of which B. the smallest of which C. the smaller of them D. smallest of that 8. Pioneer men and women endured terrible hardships, and _ A. so do their children B. neither did the children C. also the childs D. so did their children 9. Last year, Matt earned his brother, who has a better position. A. twice as much as B. twice more than C. twice as many as D. twice as more as 10. ,he would have been able to pass the exam. 120
  • 122. Chapter Eight Exercises: Test A. If he studied more B. if he were sdudying to a greater degree C. Sduying more D. Had he studied more 11. Mr.Duncan does not know the lawn mower after they had finished using it. A. where did they put B. where they did put C. where they put D. where to put 12. The facilities of the older hospital ---,-;- A. is as good or better than the new hospital B. are as good or better that the new hospital C. are as good or better than the new hospital D. are as good as or better than those of the new hospital 13. pandas eat bamboo almost exclusively, they are also carnivorous. A. Not only B. Untill C. As soon as D. Although 14. Although _ _a country illegally is risky, the alien who finds work may believe the risk worthwhile. A. when entering B. he enters C. entering D.havin g entered 15. The Andean condor glides on air currents and doesn't flap its wings__ it must do so to reach updrafts. A. becuase B. until C. that D. as if 16. __ sighting an approaching car, some drivers tend to speed up. A. When slowing down instead of B. Instead when slowing down at C. When instead of slowing down D. Instead of slowing down when 17. to England remain strong, the Channel Islanders are exempt from most British taxes. A. Although their ties B. Although tied C. Before their ties D. Tied 18. _ _ , the travlers found that their flight had been canceled because of the severe snowstorm. A. That they arrived at the airport B. As soon as arriving at the airport C. At the airport D. They arrived at the airport 19. When competing in a demolition derby, _ _ until their ccars are demolished. A. that drivers continue B. drivers must continue C. drivers continuing D. although drivers must continue 121
  • 123. Chapter Eight Exercises: Test 20. _ _ governments point with pride to icreasing mechanisation in agriculture, human and animal power still produces a significant portion of the world's food. A. Since B. Because C. So that D. While 21. , tobacco farmers had not yet felt its effect. A. Though a campaign against smoking B. That there was a campaign against smoking C. Even though there was a campaign against smoking D. There was a campaign against smoking 22. There were few settlements along the North Caroline coast_ _ many problems for seafarers. A. because the offshore barrier posed B. before posing the offshore barrier C. while posing the offshore barrier D. that the offshore barrier had posed 23. since the death of her father. A. The ancestral home of my mother abandoned B. My mother's ancestral home standing abandoned C. My mother's ancestral home has stood abandoned D. My mother's ancestral home which has stood abandoned 24. , the owner and buyer finally agreed on a price for the house. A. They had been bargaining for several weeks B. After bargaining for several weeks C. After several weeks they began bargaining D. As if bargaining for several weeks 25. _ _ of the tranquilizer, the scientist put a tag on its ear and recorded details about the animal. A. While under the effect the deer B. While being under the effect the deer C. While the deer was under the effect D. While the deer under the effect 26. He had been cutting human hairs for two years before he came to ABC 0 Darhkan. 27. Linseed oil exposed to airs forms a robbery film. ABC 0 28. The principal called the teachers' attentions to the fact that, in the coming A B C year, the school's enrollment would be almost doubled. o 122
  • 124.
  • 125. Chapter Eight Exercises: Test e 0 38. Leading scientists are often the kinds of person who have enjoyed A B e intellectual challenges all their lives. o 39. Plants of these type grow best in places where there ;s a great deal of A B e 0 shade. 40. In spite of their aloofness and independent bearing, the domisticated cat A B has come to depend upon man for food and protection. e 0 41. When Rhodesia declared their independence from England few thought A B that the new government would last even a month. e 0 42. There are almost a million people with Spanish surnames in Los Angeles A out of a total population of more than seven millions. B e D 43. In the early 1970's total employment in the United States stood at roughly A B e eighty three millions. o 44. In the 1930's more than two thousands interview of former slaves were A B conducted by members of the Federal Writers Project. e 0 45. Both as a hobbyandas a profession, phdographyhasfascinated people A B for more than hundred years. e 0 46. For all their knowledge and years' observations, astronomers have more A B questions than answers about Halley's or any other comet. e .~ 0 124
  • 126. Chapter Eight Exercises: Test 47. The tamarind is an attractive tropical tree that may grow as tall as seventy- A B C five foot. o 48. Though the dust illY heavy upon the floor, it was evident that an old house, A B its windows carelesly boarded gp, must once have been a magnificent C 0 mansion. 49. The Scots have porridge for breakfast; a wedding breakfast was held in ABC 0 her father's house. 50. Prime Minister announced the names of those who were to be named to A B C o the Cabinet. 125